Sunteți pe pagina 1din 88

Total Marks : 200

Online Prelims TEST - 7 (TEXTBOOK)


( InsightsIAS Mock Test Series for UPSC Preliminary Exam 2020 )

1 The Landmark Consumer Protection Bill, 2019 has recently received Parliamentary approval. It does
NOT cover which of the following categories of consumers?
1. Persons who obtains a good for resale or a good or service for commercial purpose
2. Persons who buy goods through teleshopping or/and multi-level marketing

Select the correct answer using the codes below.


A. 1 only
B. 2 only
C. Both 1 and 2
D. None of the above

Correct Answer : A

Answer Justification :

Justification: Key Provisions of the Bill:

Definition of consumer:

A consumer is defined as a person who buys any good or avails a service for a consideration.

What it covers?

It covers transactions through all modes including offline, and online through electronic means,
teleshopping, multi-level marketing or direct selling.

What it doesn’t?

It does not include a person who obtains a good for resale or a good or service for commercial
purpose.

2. Six consumer rights have been defined in the Bill, including the right to:

(i) be protected against marketing of goods and services which are hazardous to life and property;
(ii) be informed of the quality, quantity, potency, purity, standard and price of goods or services; (iii)
be assured of access to a variety of goods or services at competitive prices; and (iv) seek redressal
against unfair or restrictive trade practices.

Learning: Why is the bill significant?

Swift remedies: Presently Consumer only have a single point of access to justice, which is
time consuming. Additional swift executive remedies are proposed in the bill through Central
Consumer Protection Authority (CCPA).
Deterrent punishment to check misleading advertisements and adulteration of products.
Product liability provision to deter manufacturers and service providers from delivering
defective products or deficient services.
Ease of approaching Consumer Commission and Simplification of Adjudication process.
Scope for early disposal of cases through mediation.

1
Total Marks : 200
Online Prelims TEST - 7 (TEXTBOOK)
( InsightsIAS Mock Test Series for UPSC Preliminary Exam 2020 )

Q Source:
https://www.insightsonindia.com/2019/08/07/insights-daily-current-affairs-pib-07-august-2019/

2 Consider the following statements.


1. The location of the thermal equator is not identical to that of the geographic Equator.
2. The location of thermal equator is the most affected by the different amounts of total solar
insolation received by the two hemispheres of earth.

Which of the above is/are correct?


A. 1 only
B. 2 only
C. Both 1 and 2
D. None

Correct Answer : A

Answer Justification :

Justification: The location of the thermal equator is not identical to that of the geographic
Equator.

This is not due to the Level of insolation received in both hemispheres. They are virtually equal.

What creates the difference is the distribution of land and water that causes changes in the way
heat is distributed and transported across the planets.

Land dominated areas will tend to have a large variation in temperature and a higher average
temperature under the same conditions (as compared to a water dominated area).

Thus, the thermal equator lies a bit north of the geographical equator.

Q Source: AR: 11th NCERT: Fundamentals of Physical Geography

3 UNSC resolution 47 often seen in news is related to

A. USA intervention in Libya


B. Peacekeeping operations
C. Neo-colonialism in Africa
D. Dispute over Jammu and Kashmir

Correct Answer : D

Answer Justification :

Justification: Pakistan PM Imran Khan has termed the Government of India’s decision to remove

2
Total Marks : 200
Online Prelims TEST - 7 (TEXTBOOK)
( InsightsIAS Mock Test Series for UPSC Preliminary Exam 2020 )

the special status for the state of Jammu and Kashmir by modifying Article 370 as illegal since it
violates UNSC resolution 47.

What is Resolution 47?

Resolution 47 of the UNSC focuses on the complaint of the Government of India concerning
the dispute over the State of Jammu and Kashmir, that India took to the Security Council in
January 1948.

In October 1947, following an invasion by soldiers from the Pakistan Army in plainclothes and
tribesmen, the Maharaja of Kashmir, Hari Singh sought assistance from India and signed
the Instrument of Accession. After the first war in Kashmir (1947-1948), India approached the
UN Security Council to bring the conflict in Kashmir to the notice of Security Council
members.

Who were the UNSC members who oversaw the issue?

The UN Security Council increased the size of the investigating council to include six
members along with permanent members of the UNSC. Along with the five permanent
members, China, France, UK, US & Russia, non-permanent members included Argentina, Belgium,
Canada, Colombia, Syria and the Ukrainian Soviet Socialist Republic.

Learning: Both countries rejected Resolution 47.

Why India rejected?

India’s contention was that the resolution ignored the military invasion by Pakistan and
placing both nations on an equal diplomatic ground was a dismissal of Pakistan’s
aggression and the fact that the Maharaja of Kashmir, Hari Singh had signed the Instrument of
Accession.

India also objected to the Resolution’s requirement that did not allow India to retain military
presence which it believed it needed for defence.

Q Source:
https://www.insightsonindia.com/2019/08/07/insights-daily-current-affairs-pib-07-august-2019/

4 With reference to Asian Infrastructure and Investment Bank (AIIB), consider the following statements.
1. It offers financing for sustainable projects in energy, telecommunications and rural infrastructure.
2. It may participate in the underwriting of securities issued by any entity or enterprise for purposes
consistent with its purpose.

Which of the above is/are correct?


A. 1 only
B. 2 only
C. Both 1 and 2
D. None

3
Total Marks : 200
Online Prelims TEST - 7 (TEXTBOOK)
( InsightsIAS Mock Test Series for UPSC Preliminary Exam 2020 )

Correct Answer : C

Answer Justification :

Justification: According to the Articles of Agreement (AOA) of AIIB, the Bank will “provide or
facilitate financing to any member, or any agency, instrumentality or political subdivision thereof, or
any entity or enterprise operating in the territory of a member, as well as to international or
regional agencies or entities concerned with economic development of the Asia region.”

Furthermore, the AOA permits the Bank to provide financing in a variety of ways, including, inter
alia, making loans, investing in the equity capital of an enterprise, and guaranteeing, whether as
primary or secondary obligor, in whole or in part, loans for economic development.

In addition, the Bank may underwrite, or participate in the underwriting of, securities issued by any
entity or enterprise for purposes consistent with its purpose.

Other areas of Bank finance include agriculture development, water supply and sanitation,
environmental protection, and urban development and logistics.

Q Source: Revision: Ramesh Singh: Indian Economy:


https://www.aiib.org/en/about-aiib/who-we-are/financing-operations/index.html

5 The eastern and western boundaries of the Pacific Ocean experience frequent earthquakes mainly due
to
1. Their margins coinciding with the plate margins.
2. The presence of high mountain stretches along the continental belts

Which of the above is/are correct?


A. 1 only
B. 2 only
C. Both 1 and 2
D. None of the above

Correct Answer : A

Answer Justification :

Justification: The region in question is the Ring of Fire.

The Ring of Fire is a direct result of plate tectonics/ movement and collisions of lithospheric plates.

4
Total Marks : 200
Online Prelims TEST - 7 (TEXTBOOK)
( InsightsIAS Mock Test Series for UPSC Preliminary Exam 2020 )

Since the margins coincide with the plate margins, the continous tectonic movements cause
earthquakes.

This region is also prone to high volcanism.

Q Source: NCERT 11th Geography: Fundamentals of Physical Geography

6 The Lok Sabha has passed the Surrogacy (Regulation) Bill, 2019 by a voice vote. Consider the
following about the provisions of the bill.
1. The bill bans commercial surrogacy.
2. The intending couple should have a ‘certificate of essentiality’ and a ‘certificate of eligibility’ issued
by the appropriate authority.
3. The surrogate mother must possess a certificate of medical and psychological fitness for surrogacy.
4. The surrogate mother will have an option to withdraw from the surrogacy anytime before the
delivery of the child.

Select the correct answer using the codes below.


A. 1, 2 and 3 only
B. 2, 3 and 4 only
C. 1, 3 and 4 only
D. 1, 2 and 4 only

Correct Answer : A

Answer Justification :

Justification: Key features of the bill:

5
Total Marks : 200
Online Prelims TEST - 7 (TEXTBOOK)
( InsightsIAS Mock Test Series for UPSC Preliminary Exam 2020 )

Regulation of surrogacy: The Bill prohibits commercial surrogacy, but allows altruistic
surrogacy.

Purposes for which surrogacy is permitted: Surrogacy is permitted when it is: (i) for intending
couples who suffer from proven infertility; (ii) altruistic; (iii) not for commercial purposes; (iv) not
for producing children for sale, prostitution or other forms of exploitation; and (v) for any condition
or disease specified through regulations.

Eligibility criteria for intending couple: The intending couple should have a ‘certificate of
essentiality’ and a ‘certificate of eligibility’ issued by the appropriate authority.

A certificate of essentiality will be issued upon fulfilment of the following conditions: (i) a
certificate of proven infertility of one or both members of the intending couple from a District
Medical Board; (ii) an order of parentage and custody of the surrogate child passed by a
Magistrate’s court; and (iii) insurance coverage for a period of 16 months covering postpartum
delivery complications for the surrogate.

The certificate of eligibility to the intending couple is issued upon fulfilment of the
following conditions: (i) the couple being Indian citizens and married for at least five years; (ii)
between 23 to 50 years old (wife) and 26 to 55 years old (husband); (iii) they do not have any
surviving child (biological, adopted or surrogate); this would not include a child who is mentally or
physically challenged or suffers from life threatening disorder or fatal illness; and (iv) other
conditions that may be specified by regulations.

Eligibility criteria for surrogate mother: To obtain a certificate of eligibility from the
appropriate authority, the surrogate mother has to be: (i) a close relative of the intending couple;
(ii) a married woman having a child of her own; (iii) 25 to 35 years old; (iv) a surrogate only once in
her lifetime; and (v) possess a certificate of medical and psychological fitness for surrogacy.
Further, the surrogate mother cannot provide her own gametes for surrogacy.

Appropriate authority: The central and state governments shall appoint one or more appropriate
authorities within 90 days of the Bill becoming an Act. The functions of the appropriate authority
include; (i) granting, suspending or cancelling registration of surrogacy clinics; (ii) enforcing
standards for surrogacy clinics; (iii) investigating and taking action against breach of the provisions
of the Bill; (iv) recommending modifications to the rules and regulations.

Parentage and abortion of surrogate child: A child born out of a surrogacy procedure will be
deemed to be the biological child of the intending couple. An abortion of the surrogate child
requires the written consent of the surrogate mother and the authorisation of the appropriate
authority. This authorisation must be compliant with the Medical Termination of Pregnancy Act,
1971. Further, the surrogate mother will have an option to withdraw from surrogacy before
the embryo is implanted in her womb.

Q Source: As mentioned above

7 The higher the amount of Biochemical Oxygen Demand (BOD) in a certain segment of water
1. Higher is the amount of organic matter
2. Higher could be the level of pollution in the water

6
Total Marks : 200
Online Prelims TEST - 7 (TEXTBOOK)
( InsightsIAS Mock Test Series for UPSC Preliminary Exam 2020 )

Which of the above is/are correct?


A. 1 only
B. 2 only
C. Both 1 and 2
D. None

Correct Answer : C

Answer Justification :

Justification: Biochemical Oxygen Demand or Biological Oxygen Demand, is a measurement of the


amount of dissolved oxygen (DO) that is used by aerobic microorganisms when decomposing
organic matter in water.

Biochemical Oxygen Demand is an important water quality parameter because it provides an index
to assess the effect discharged wastewater will have on the receiving environment. The higher the
BOD value, the greater the amount of organic matter or “food” (read nutrients) available for oxygen
consuming bacteria. So, both 1 and 2 are correct.

Learning: If the rate of DO consumption by bacteria exceeds the supply of DO from aquatic plants,
algae photosynthesis or diffusing from air, unfavourable conditions occur.

Depletion of DO causes stress on aquatic organisms, making the environment unsuitable for life.
Further, dramatic depletion can lead to hypoxia or anoxic environments. BOD is also used
extensively for wastewater treatment, as decomposition of organic waste by microorganisms is
commonly used for treatment.

Q Source: Page 407: Chemistry Unit 14: XIth NCERT

8 Consider the following statements.


The river Kosi
1. is a trans-national river that originates from Tibet
2. flows through Nepal and the plains of North Bihar

Select the correct answer using the codes below.


A. 1 only
B. 2 only
C. Both 1 and 2
D. None of the above

Correct Answer : C

Answer Justification :

Justification and Learning: Union Government has approved Rs 4,900 crore Kosi-Mechi
Interlinking project for interlinking of Kosi and Mechi rivers of Bihar. This is the second

7
Total Marks : 200
Online Prelims TEST - 7 (TEXTBOOK)
( InsightsIAS Mock Test Series for UPSC Preliminary Exam 2020 )

major river interlinking project in the country to be approved by Central Government after
the Ken-Betwa project in Madhya Pradesh.

Need and significance:

The river Kosi is an international river originating from Tibet and flowing through Nepal
in Himalayan Mountains and the lower portion through plains of North Bihar.

To overcome the acute problem of shifting of course of Kosi river, heavy sediment load,
flooding etc. and to alleviate the severe suffering of the people of Bihar, the then His
Majesty’s Government of Nepal and The Government of India signed an agreement on 25th
April 1954 for implementation of Kosi project. The present proposal is an extension
of Eastern Kosi Main Canal (EKMC) system upto river Mechi, a tributary of river
Mahananda.

The aim of extension of EKMC upto Mechi river is mainly to provide irrigation benefits to
the water scarce Mahananda basin command in the districts of Araria, Kishanganj,
Purnea and Katihar during kharif season depending upon the pondage available in Hanuman
Nagar barrage.

This intrastate link scheme will thus transfer part of surplus water of Kosi basin to
Mahananda basin. In view of irrigation benefit from the link canal, the project is fully
justified.

Q Source:
https://www.insightsonindia.com/2019/08/06/insights-daily-current-affairs-pib-06-august-2019/

9 Consider the following statements.


Quick Reaction Surface-to-Air missiles (QRSAM)
1. has been developed indigenously by DRDO
2. has been developed as all weather and all terrain missile.
3. can hit targets in the range of 25-30 KMs and low-flying objects.

Select the correct answer using the codes below.


A. 1 only
B. 2 and 3 only
C. 1 and 3 only
D. 1, 2 and 3

Correct Answer : D

Answer Justification :

8
Total Marks : 200
Online Prelims TEST - 7 (TEXTBOOK)
( InsightsIAS Mock Test Series for UPSC Preliminary Exam 2020 )

Justification: DRDO has successfully test-fired indigenously developed Quick Reaction Surface-
to-Air missiles (QRSAM) from a test range off the Odisha coast.

About QRSAM:

1. It has been developed to replace the ‘Akash’ missile defence system, and has 360-degree
coverage.

2. It uses solid fuel propellant and has a strike range of 25-30 km with capability of hitting
multiple targets.

3. It is capable of hitting the low flying objects.

Q Source:
https://www.insightsonindia.com/2019/08/05/insights-daily-current-affairs-pib-05-august-2019/

10 One of the world's richest fishing grounds situated around the coast of Iceland. Which of the
following factors explain it?
1. Coastal areas adjoin high density of warm currents
2. Abundance of phytoplankton
3. Deep marine trenches are found there

Which of the above is/are correct?


A. 1 only
B. 3 only
C. 2 and 3 only
D. 2 only

Correct Answer : D

Answer Justification :

Learning: The following conditions are good for fisheries production & catch:

Coastal upwelling of nutrient rich colder water


Abundance of phytoplankton
Meeting of warm and cold currents
Shallow continental shelves in cold regions

It is exemplified by the fact that one of the world's richest fishing grounds situated around the coast
of Iceland.

Q Source: NCERT 11th Geography: Fundamentals of Physical Geography

9
Total Marks : 200
Online Prelims TEST - 7 (TEXTBOOK)
( InsightsIAS Mock Test Series for UPSC Preliminary Exam 2020 )

11 The Department of Bio-technology (DBT) plans to scan nearly 20,000 Indian genomes over the next
five years, in a two-phase exercise. Consider the following statements about it.
1. The data generated by the project will be accessible only to certain licensed public institutions to
avoid the misuse of genome data.
2. Genome screening of “diseased individuals” will not be undertaken under the project.

Select the correct answer using the codes below.


A. 1 only
B. 2 only
C. Both 1 and 2
D. None of the above

Correct Answer : D

Answer Justification :

Justification: The Department of Biotechnology (DBT) plans to scan nearly 20,000 Indian genomes
over the next five years, in a two-phase exercise, and develop diagnostic tests that can be used to
test for cancer.

Key facts:

1. The first phase involves sequencing the complete genomes of nearly 10,000 Indians from all
corners of the country and capture the biological diversity of India.

2. In the next phase, about 10,000 “diseased individuals” would have their genomes sequenced.
These vast troves of data sets would be compared using machine learning techniques to
identify genes that can predict cancer risk, as well as other diseases that could be
significantly influenced by genetic anomalies.

Significance:

1. The data generated would be accessible to researchers anywhere for analysis. This would be
through a proposed National Biological Data Centre envisaged in a policy called the
‘Biological Data Storage, Access and Sharing Policy’, which is still in early stages of
discussion.

2. As the genetic landscape differs across the world, it is necessary that genetic data is shared in
order to derive greater knowledge from information and serve the purpose of enabling better
treatment outcomes.

3. The GenomeIndia initiative will pave the way for identifying genes and genetic variations for
common diseases, treating Mendelian disorders, enabling the transformation of the Precision

10
Total Marks : 200
Online Prelims TEST - 7 (TEXTBOOK)
( InsightsIAS Mock Test Series for UPSC Preliminary Exam 2020 )

Medicine landscape in India, and thus improving the healthcare of the general population in
our country.

Need for genome sequencing:

Ever since the human genome was first sequenced in 2003, it opened a fresh perspective on
the link between disease and the unique genetic make-up of each individual.

Nearly 10,000 diseases — including cystic fibrosis, thalassemia — are known to be the result of
a single gene malfunctioning. While genes may render some insensitive to certain
drugs, genome sequencing has shown that cancer too can be understood from the
viewpoint of genetics, rather than being seen as a disease of certain organs.

Q Source: As mentioned above

12 How UV radiation is harmful to terrestrial life?


1. It accelerates ageing of skin in humans.
2. It can kill phytoplankton in large numbers.
3. Plant proteins are adversely affected by UV radiation.
4. It may lead to harmful mutation of plant cells.

Select the correct answer using the codes below.


A. 1 and 4 only
B. 2 and 3 only
C. 1, 2 and 4 only
D. 1, 2, 3 and 4

Correct Answer : D

Answer Justification :

Justification: Statement 1: With the depletion of ozone layer, more UV radiation filters into
troposphere. UV radiations lead to ageing of skin, cataract, sunburn and skin cancer in humans.

Statement 2: Many phytoplankton cannot tolerate the bright, unfiltered solar radiation at the
ocean's surface and are impaired especially by the high-energy, short-wavelength radiation in the
UV range of the spectrum.

Statement 3 and 4: UV-B impairs photosynthesis in many species. Overexposure to UV-B reduces
size, productivity, and quality in many of the crop plant species. UV-B increases plants’
susceptibility to disease by sometimes causing mutations or damaging plant proteins.

Learning: It also increases evaporation of surface water through the stomata of the leaves and
decreases the moisture content of the soil. Increase in UV radiations damage paints and fibres,
causing them to fade faster.

You can read more here http://earthobservatory.nasa.gov/Features/UVB/uvb_radiation2.php

11
Total Marks : 200
Online Prelims TEST - 7 (TEXTBOOK)
( InsightsIAS Mock Test Series for UPSC Preliminary Exam 2020 )

Q Source: Page 406: Chemistry Unit 14: XIth NCERT

13 Indian individuals who have received the Ramon Magsasay Award in the field of Journalism,
Literature, and the Creative Communication Arts include
1. Satyajit Ray
2. Pandit Ravi Shankar
3. Palagummi Sainath

Select the correct answer using the codes below.


A. 3 only
B. 1 and 3 only
C. 2 only
D. 1, 2 and 3

Correct Answer : D

Answer Justification :

Justification: You can go through the list of award winners from India at the Q Source. Try going
through the prominent names in each category.

The categories for which it is awarded are:

GS Government Service
PS Public Service
CL Community Leadership
JLCCA Journalism, Literature, and the Creative Communication Arts
PIU Peace and International Understanding
EL Emergent Leadership

Source: https://en.wikipedia.org/wiki/List_of_Ramon_Magsaysay_Award_winners

14 UNIDO and the Global Environment Facility (GEF) are partnering with Ministry of New and
Renewable Energy, India, to support capacity building and skill development of technical manpower
in the Concentrated Solar Thermal Energy Technologies (CST). What are the benefits of CST?
1. It will be able to replace conventional fossil fuels such as coal, diesel and furnace oil in certain
industrial applications
2. It will allow us to save costs and emissions in the industrial process heat applications

Select the correct answer using the codes below.


A. 1 only
B. 2 only
C. Both 1 and 2
D. None of the above

12
Total Marks : 200
Online Prelims TEST - 7 (TEXTBOOK)
( InsightsIAS Mock Test Series for UPSC Preliminary Exam 2020 )

Correct Answer : C

Answer Justification :

Justification: UNIDO and National Institute of Solar Energy to initiate a skill development
programme for different levels of beneficiaries in the solar thermal energy sector.

NISE and UNIDO will engage national and international experts to bring the best practices by
developing specialized training material.

Background: The agreement is part of the ongoing MNRE-GEF-UNIDO project implemented jointly
by UNIDO and to support capacity building and skill development of technical manpower in the
Concentrated Solar Thermal Energy Technologies (CST) which are being used to replace
conventional fossil fuels e.g. coal, diesel, furnace oil etc. and save costs and emissions in the
industrial process heat applications.

Different concentrating technologies have been developed or are currently under development for
various commercial and industrial applications. For industrial processes where temperatures above
80°C are required, concentrating solar collectors such as parabolic trough or dish collectors, non-
imaging concentrators or a Linear Fresnel system are required to be used.

The GEF-UNIDO’s project is designed to complement MNRE’s support programme by helping to


remove barriers associated with CST technology, its awareness, capacity building, market and
financial barriers. The duration of the project is from January 2015 to December 2019. In addition,
the initiative aims to provide technology application information packages and standardization of
CST performance measurement.

The project strategy builds on the existing favorable framework for solar thermal in India. Factors
in favor of the project include the high commitment by the government to the development of its
solar thermal industry, and significant interest by the industrial sector to reduce its reliance on
fossil fuels. Primary target beneficiaries of the project are energy policy-making and implementing
institutions, primarily MNRE, MSME, IREDA, industrial unit owners (end beneficiaries), CS
manufacturers, designers, installers, training institutes, energy professionals and service providers
and the financial sector.

Q Source: AR
https://www.insightsonindia.com/2019/08/08/insights-daily-current-affairs-pib-08-august-2019/

https://open.unido.org/api/documents/12718335/download/Project%20Factsheet

15 The Quit India movement (QIM) was started by Mahatma Gandhi in 1942. Consider the following
statements associated with the movement and its immediate consequences.
1. The Indian National Congress (INC) was declared an unlawful association.
2. The senior INC leaders were imprisoned without trial within hours of Gandhi’s speech that declared
the start of the QIM.

Select the correct answer using the codes below.


A. 1 only
13
Total Marks : 200
Online Prelims TEST - 7 (TEXTBOOK)
( InsightsIAS Mock Test Series for UPSC Preliminary Exam 2020 )

B. 2 only
C. Both 1 and 2
D. None of the above

Correct Answer : C

Answer Justification :

Justification: The Quit India movement was started by Mahatma Gandhi in 1942 but drew
protests from the All-India Congress Committee demanding what Gandhi called was “An Orderly
British Withdrawal” from India. This forced the British to act immediately and soon all the senior
INC leaders were imprisoned without trial within hours of Gandhi’s speech.

Other key facts:

1. Several national leaders like Mahatma Gandhi, Abdul Kalam Azad, Jawaharlal Nehru and
Sardar Vallabhbhai Patel were arrested.

2. The Congress was declared an unlawful association, leaders were arrested and its offices
all over the country were raided and their funds were frozen.

3. The first half of the movement was peaceful with demonstrations and processions. The
peaceful protest was carried till Mahatma Gandhi’s release.

4. The second half of the movement was violent with raids and setting fire at post offices,
government buildings and railway stations. Lord Linlithgow adopted the policy of violence.

5. The Viceroy’s Council of Muslims, Communist Party and Americans supported


Britishers.

Q Source:
https://www.insightsonindia.com/2019/08/08/insights-daily-current-affairs-pib-08-august-2019/

16 Osmosis is a driving force, i.e. assists the mechanism, for which of these diseases?

A. HIV AIDS
B. Cholera
C. Tuberculosis
D. All of the above

Correct Answer : B

14
Total Marks : 200
Online Prelims TEST - 7 (TEXTBOOK)
( InsightsIAS Mock Test Series for UPSC Preliminary Exam 2020 )

Answer Justification :

Justification & Learning: In Osmosis, water will move from its region of lower chemical potential
(or concentration) to its region of lower chemical potential until equilibrium is reached.

At equilibrium the two chambers should have the same water potential. For e.g. If you put a
carrot in salty water, the salt water will "draw" the water from inside the carrot thus drying it.
Cholera would not be possible without osmosis. The choleric bacteria populate in our
intestines and begin to reverse the intestinal cells’ ionic orientation. In other words, it
changes the way ions and, subsequently, water is transported in our intestines.
when our ions’ orientations are switched, the intestinal cells are no longer able to absorb
water into the body. It actually flows out. Now osmosis happens in the other direction and
water moves from our intestinal cells into our intestines. This is what causes cholera’s deadly
watery diarrhoea.
This is why cholera can kill you so quickly, because it does not rely on how much water you
consume.

Q Source: AR: Page 180: Biology: XIth NCERT

17 The Environment Ministry has unveiled a draft Environmental and Social Management Framework
(ESMF). Consider the following about it.
1. The draft is part of a World Bank-funded project.
2. The draft plan will dictate how prospective infrastructure projects situated along the coast ought to
be assessed before they can apply for clearance.
3. It lays out guidelines out for coastal States to adopt when they approve and regulate projects in
coastal zones.

Select the correct answer using the codes below.


A. 1 only
B. 2 and 3 only
C. 1 and 2 only
D. 1, 2 and 3

Correct Answer : D

Answer Justification :

Justification: Key highlights:

1. The draft is part of a World Bank-funded project.

2. The draft plan will dictate how prospective infrastructure projects situated along the
coast ought to be assessed before they can apply for clearance.

3. It lays out guidelines out for coastal States to adopt when they approve and regulate

15
Total Marks : 200
Online Prelims TEST - 7 (TEXTBOOK)
( InsightsIAS Mock Test Series for UPSC Preliminary Exam 2020 )

projects in coastal zones.

4. The plan describes how “environmental and social aspects” ought to be integrated into the
planning, design, implementation of projects.

5. It says, projects should strive to avoid or minimise impacts on cultural properties and
natural habitats, compensate any loss of livelihood or assets, adopt higher work
safety standards, occupational and community health and safety.

Background:

The project seeks to assist the Government of India in enhancing coastal resource efficiency
and resilience, by building collective capacity (including communities and decentralised
governance) for adopting and implementing integrated coastal management approaches.

So far three coastal States, namely Gujarat, Odisha and West Bengal, have prepared Integrated
Coastal Zone Management Plans with support from the World Bank.

Such plans would be prepared for the selected coastal stretches in other States/UT, the project
notes.

Learning: The key activities proposed for coastal zone development that consist of investments by
States include:

1. Mangrove afforestation/shelter beds.

2. Habitat conservation activities such as restoration of sea-grass meadows.

3. Eco-restoration of sacred groves.

4. Development of hatcheries.

5. Rearing/rescue centres for turtles and other marine animals.

6. Creation of infrastructure for tourism.

7. Restoration and recharge of water bodies.

8. Beach cleaning and development.

16
Total Marks : 200
Online Prelims TEST - 7 (TEXTBOOK)
( InsightsIAS Mock Test Series for UPSC Preliminary Exam 2020 )

9. Other small infrastructure facilities.

Q Source:
https://www.insightsonindia.com/2019/08/08/insights-daily-current-affairs-pib-08-august-2019/

18 Pashmina goats are generally domesticated and reared by which of these nomadic communities?

A. Zira
B. Changpa
C. Nubo
D. Dansa

Correct Answer : B

Answer Justification :

Justification: Changthangi or Pashmina goat:

1. It is a special breed of goat indigenous to the high altitude regions of Ladakh in Jammu
and Kashmir.

2. They are raised for ultra-fine cashmere wool, known as Pashmina once woven.

3. These goats are generally domesticated and reared by nomadic communities called the
Changpa in the Changthang region of Greater Ladakh.

4. The Changthangi goats have revitalized the economy of Changthang, Leh and Ladakh region.

The Leh district of Greater Ladakh produces around 30 000 kg of pashmina fibre which is harvested
from about 0.15 million Changthangi goats reared by the Changpa nomads in Changthang region of
Greater Ladakh.

Q Source:
https://www.cambridge.org/core/journals/animal-genetic-resources-resources-genetiques-animales-r
ecursos-geneticos-animales/article/changthangi-goats-a-rich-source-of-pashmina-production-in-
ladakh/D17273291DFC936E266CBF32B559ED71#

https://www.insightsonindia.com/2019/08/03/insights-daily-current-affairs-pib-03-august-2019/

19 If you enclose a healthy plant in polythene bag, droplets of water start forming inside the bag. This
can be attributed to
1. Photosynthesis

17
Total Marks : 200
Online Prelims TEST - 7 (TEXTBOOK)
( InsightsIAS Mock Test Series for UPSC Preliminary Exam 2020 )

2. Transpiration

Which of the above is/are correct?


A. 1 only
B. 2 only
C. Both 1 and 2
D. None of the above

Correct Answer : B

Answer Justification :

Learning: Water is transient in plants. Less than 1 per cent of the water reaching the leaves is used
in photosynthesis and plant growth. Most of it is lost through the stomata in the leaves. This water
loss is known as transpiration.

When plants lose water, it will be shown as moisture on the inside of the bag (in which you have
covered the plant).

Transpiration is important in many ways for a plant. It cools them and helps in water transportation
to great heights in trees.

Q Source: Page 187: Biology: XIth NCERT

20 This plant growth regulator breaks seed and bud dormancy; helps in the process of stimulating or
regulating the ripening of fruits. It is

A. Ethylene
B. Cytokinin
C. Gibrellin
D. Auxin

Correct Answer : A

Answer Justification :

Learning: Ethylene is a simple gaseous Plant growth regulator (PGR). It is synthesised in large
amounts by tissues undergoing senescence and ripening fruits.

Influences of ethylene on plants include horizontal growth of seedlings, swelling of the axis etc.
Ethylene promotes abscission of plant organs especially of leaves and flowers.

Ethylene is highly effective in fruit ripening. It enhances the respiration rate during ripening of the
fruits. This rise in rate of respiration is called respiratory climactic.

Q Source: Chapter 15: Biology: XIth NCERT

18
Total Marks : 200
Online Prelims TEST - 7 (TEXTBOOK)
( InsightsIAS Mock Test Series for UPSC Preliminary Exam 2020 )

21 Consider the following statements.


1. There is no code of Conduct for members of either house of Parliament in India.
2. Other than the UK and Canada, no other country prescribes a code of conduct for the
Parliamentarians.

Select the correct answer using the codes below.


A. 1 only
B. 2 only
C. Both 1 and 2 only
D. None of the above

Correct Answer : D

Answer Justification :

Background: Code of conduct for high constitutional functionaries and representatives of the
people have been discussed for long. A code for Union ministers was adopted in 1964, and state
governments were advised to adopt it as well.

Justification: Statement 1: A Code of Conduct for members of Rajya Sabha has been in force
since 2005; there is no such code for Lok Sabha.

Key recommendations for the code of conduct are:

1. Prohibit MPs from misusing the power and immunities they get.

2. An MP should avoid conflict between a private and a public interest.

3. No parliamentarian should be allowed to vote on those questions in the House, in which


he/she has a vested interest.

4. Amend the Constitution to ensure a minimum of 110 days of sitting in a legislature having
more than 100 members, and 90-50 days of sitting in Houses with less than 100 members
depending on the size of the State involved.

5. The filing by legislators of a statement of income, assets and liabilities, and an indication of
changes in these figures over time.

6. Punishment of members by admonition, reprimand, censure or withdrawal from the House in


case of violations or breach of the code of conduct.

7. Automatic suspension from the House of any member involved in offences of grave

19
Total Marks : 200
Online Prelims TEST - 7 (TEXTBOOK)
( InsightsIAS Mock Test Series for UPSC Preliminary Exam 2020 )

misconduct.

Statement 2: In other countries:

1. In the UK, a code of conduct for MPs was “prepared pursuant to the Resolution of the House
of 19 July 1995”.

2. The Canadian House of Commons has a Conflict of Interest and Ethics Commissioner with
powers to examine violations of the Conflict of Interest Code at the request of another
Member or by Resolution of the House or on his own initiative.

3. Germany has had a Code of Conduct for members of the Bundestag since 1972.

4. The US has had a Code since 1968.

5. Pakistan has a Code of Conduct for members of the Senate.

Q Source: Recently in news

22 Even when Earth gets nearest to the Sun at perihelion, the effects of this increased solar insolation
are not felt on a large scale on earth. This can be due to

A. Increase in cloudiness in major continental regions


B. Reduction in solar discharge
C. Due to circulation of ocean currents and redistribution of heat through trade winds
D. Adiabetic wind flow patterns

Correct Answer : C

Answer Justification :

Justification: This variation in the solar output does not have a large effect on daily weather
changes on earth at a certain point.

Distribution of land and sea, heat transfer by way of sea and land breezes etc redistribute the
increased insolation so that the effects are not felt on a large scale. So, C is correct.

Solar discharge does affect weather patterns, but it is not relevant here. So, B is wrong.

Adiabetic wind flow patterns are found in mountain regions. So, D is wrong.

Increase in cloudiness cannot happen in all major regions due to lack of evaporation from water

20
Total Marks : 200
Online Prelims TEST - 7 (TEXTBOOK)
( InsightsIAS Mock Test Series for UPSC Preliminary Exam 2020 )

bodies at all such locations. More cloudiness is the reason why sub-tropical regions are hotter than
tropical regions despite getting less solar insolation. So, A is not relevant here.

Q Source: Page 79: Fundamentals of Physical Geography: 11th NCERT

23 Which of the following intervention(s) have been made by the government to push the production of
ethanol based fuels in the country?
1. Enabling lignocellulosic route for ethanol procurement
2. Significantly cutting down the administered price of C-heavy molasses-based ethanol

Select the correct answer using the codes below.


A. 1 only
B. 2 only
C. Both 1 and 2
D. None of the above

Correct Answer : A

Answer Justification :

Justification: S1: Earlier, in India, ethanol was mainly sourced from molasses feedstock (cane-
sugar leftovers), but this was barely sufficient to meet the current demand. The government allowed
cellulosic productions sometime ago.

Lignocellulosic ethanol (bioethanol) is one of the most important sources of advanced biofuels
owing to its promising feedstock availability and low production costs.

Cellulosic ethanol is ethanol (ethyl alcohol) produced from cellulose (the stringy fiber of a plant)
rather than from the plant's seeds or fruit. It is a biofuel produced from grasses, wood, algae, or
other plants.

S2: Cane-based ethanol can be produced three different ways — directly from cane juice, from B-
grade and C-grade molasses.

Ethanol has been largely produced from C-heavy molasses in India, as production from the other
two varieties wasn’t remunerative enough in the absence of any stable policy regime.

The Centre sometime ago marginally reduced the price of ethanol produced from C-heavy or C-
grade molasses ( from Rs 43.70 to Rs 43.46) and also fixed a price for ethanol produced from B-
heavy molasses and sugarcane juice for the first time. This is to motivate sugar mills to produce
more ethanol so that they can focus on producing not only sugar, but also remunerative ethanol.

This would have the double advantage of: a) cutting down the oversupply of sugar and thus
arresting the crash in prices, and b) increase availability of ethanol.

Read more here


https://www.business-standard.com/article/economy-policy/making-ethanol-under-new-method-could
-solve-india-s-sugar-surplus-problem-118062900162_1.html

21
Total Marks : 200
Online Prelims TEST - 7 (TEXTBOOK)
( InsightsIAS Mock Test Series for UPSC Preliminary Exam 2020 )

Learning: The major interventions include administrative price mechanism for


ethanol, simplifying the procurement procedures of OMCs, amending the provisions of
Industries (Development & Regulation) Act, 1951 and enabling lignocellulosic route for
ethanol procurement.

1. The Government approved the National Policy on Biofuels-2018 in June 2018. The policy
has the objective of reaching 20% ethanol-blending and 5% biodiesel-blending by the
year 2030.

2. Among other things, the policy expands the scope of feedstock for ethanol production
and has provided for incentives for production of advanced biofuels.

3. The Government has also increased the price of C-heavy molasses-based ethanol.

Outcomes:

1. These interventions of the Government of India have shown positive results.

2. Ethanol blending in petrol has increased from 38 crore litres in the ethanol supply year
2013-14 to an estimated 141 crore litres in the ethanol supply year 2017-18.

3. Bio-diesel blending in the country started from 10th August, 2015 and in the year 2018-19, Oil
Marketing Companies have allocated 7.6 crore litres of biodiesel.

4. Oil PSUs are also planning to set up 12 Second Generation (2G) Bio-refineries to augment
ethanol supply and address environmental issues arising out of burning of agricultural
biomass.

Q Source: As mentioned above

24 Factors affecting ocean salinity include


1. Freezing and thawing of ice in Polar Regions
2. Wind flow over the ocean
3. Areal extent of the Ocean

Select the correct answer using the codes below.


A. 1 and 2 only
B. 2 only
C. 1 and 3 only
D. 1, 2 and 3

22
Total Marks : 200
Online Prelims TEST - 7 (TEXTBOOK)
( InsightsIAS Mock Test Series for UPSC Preliminary Exam 2020 )

Correct Answer : D

Answer Justification :

Justification: Statement 1: The salinity of water in the surface layer of oceans depend mainly on
evaporation and precipitation.

Surface salinity is greatly influenced in coastal regions by the fresh water flow from rivers, and in
polar regions by the processes of freezing and thawing of ice.

Also, Baltic Sea records low salinity due to influx of river waters in large quantity.

Statement 2: Wind, also influences salinity of an area by transferring water to other areas.

The ocean currents contribute to the salinity variations. Salinity, temperature and density of water
are interrelated. Hence, any change in the temperature or density influences the salinity of an area.

The North Sea, in spite of its location in higher latitudes, records higher salinity due to more saline
water brought by the North Atlantic Drift.

Statement 3: The salinity variation in the Pacific Ocean is mainly due to its shape and larger areal
extent. Salinity decreases on the western parts of the northern hemisphere because of the influx of
melted water from the Arctic region.

25 With reference to Community Reserves in India, consider the following statements.


1. They can act as buffer zones to or connectors and migration corridors between established
protected areas.
2. They are always government owned.
3. There is currently a proposal by the Ministry of Environment, Forests and Climate Change
(MoEFCC) to give statutory backing to community reserves.

Select the correct answer using the codes below.


A. 1 and 3 only
B. 1 only
C. 1, 2 and 3 only
D. 2 and 3 only

Correct Answer : B

Answer Justification :

Justification: Statement 1: Conservation reserves and community reserves in India are terms
denoting protected areas of India which typically act as buffer zones to or connectors and migration
corridors between established national parks, wildlife sanctuaries and reserved and protected
forests of India.

Such areas are designated as conservation areas if they are uninhabited and completely owned by
23
Total Marks : 200
Online Prelims TEST - 7 (TEXTBOOK)
( InsightsIAS Mock Test Series for UPSC Preliminary Exam 2020 )

the Government of India but used for subsistence by communities, and community areas if part of
the lands are privately owned. Administration of such reserves would be through local people and
local agencies like the gram panchayat, as in the case of communal forests.

Community reserves are the first instances of private land being accorded protection under the
Indian legislature. It opens up the possibility of communally owned for-profit wildlife resorts, and
also causes privately held areas under non-profit organizations like land trusts to be given
protection.

Statement 3: These protected area categories were first introduced in the Wildlife (Protection)
Amendment Act of 2002 − the amendment to the Wildlife Protection Act of 1972.

These categories were added because of reduced protection in and around existing or proposed
protected areas due to private ownership of land, and land use.

Learning: These categories roughly correspond to IUCN Category V (conservation reserves) and VI
(community reserves) protected areas.

In 2012, Rajasthan government in India declared "Jawai Bandh forests" as a conservation reserve
forest. Jawai Bandh forest is situated in Pali district and it is in close proximity of Kumbalgarh
Sanctuary.keshopur chamb gurdaspur (Punjab) conservation reserve India's first community
reserve [4].Keshopur chamb,Gurdaspur (Punjab) is India's first community reserve.

Q Source: http://wiienvis.nic.in/Database/cri_8228.aspx

AR: https://www.insightsonindia.com/2019/08/10/insights-daily-current-affairs-pib-10-august-2019/

26 Climatically and geographically, which among the following is the largest system?

A. Ecosystem
B. Population
C. Community
D. Biome

Correct Answer : D

Answer Justification :

Learning: Ecosystem is the groups of organisms from all biological domains in conjunction with the
physical (abiotic) environment.

Biome is the continental scale (climatically and geographically contiguous areas with similar
climatic conditions) grouping of ecosystems.

Biosphere or Ecosphere is all life plus the physical (abiotic) environment.

24
Total Marks : 200
Online Prelims TEST - 7 (TEXTBOOK)
( InsightsIAS Mock Test Series for UPSC Preliminary Exam 2020 )

Q Source: Page 128: Fundamentals of Physical Geography: 11th NCERT

27 Convention on International Trade in Endangered Species of Wild Fauna and Flora (CITES) concerns
species of which of the following kinds?
1. Species that are in danger of extinction
2. Species that are not threatened with extinction but that might suffer a serious decline in number if
trade is not restricted
3. Species protected in at least one country that is a CITES member states and that has petitioned
others for help in controlling international trade in that species

Select the correct answer using the codes below.


A. 1 only
B. 1 and 2 only
C. 2 and 3 only
D. 1, 2 and 3

Correct Answer : D

Answer Justification :

Justification: CITES classifies plants and animals according to three categories, or appendices,
based on how threatened. They are.

25
Total Marks : 200
Online Prelims TEST - 7 (TEXTBOOK)
( InsightsIAS Mock Test Series for UPSC Preliminary Exam 2020 )

1. Appendix I: It lists species that are in danger of extinction. It prohibits commercial trade of these
plants and animals except in extraordinary situations for scientific or educational reasons.

2. Appendix II species: They are those that are not threatened with extinction but that might suffer
a serious decline in number if trade is not restricted. Their trade is regulated by permit.

3. Appendix III species: They are protected in at least one country that is a CITES member states
and that has petitioned others for help in controlling international trade in that species.

Learning: Convention on International Trade in Endangered Species of Wild Fauna and Flora
(CITES):

It is an International agreement to regulate worldwide commercial trade in wild animal


and plant species.
It restricts trade in items made from such plants and animals, such as food, clothing,
medicine, and souvenirs.
It was signed on March 3, 1973 (Hence world wildlife day is celebrated on march 3).
It is administered by the United Nations Environment Programme (UNEP).
Secretariat — Geneva (Switzerland).
CITES is legally binding on state parties to the convention, which are obliged to adopt their
own domestic legislation to implement its goals.

Q Source:
https://www.insightsonindia.com/2019/08/09/insights-daily-current-affairs-pib-09-august-2019/

28 'Podzolisation' leading to podsolic soils is a phenomenon most noticed in which of these forest types?

A. Tropical evergreen
B. Moist deciduous
C. Boreal forests
D. Tundra regions

Correct Answer : C

Answer Justification :

Justification: Most of the soils in Boreal forests are strongly leached as a result of centuries of
leaching and somewhat impoverished parent materials.

The soils are generally strongly acid and the main soil process under this dominantly leaching
climate is 'podzolisation' leading to podsolic soils.

The extreme climate coupled with the strong soil acidity means that relatively few organisms exist
in the soil and the turnover of organic matter is slow.

The species present are able to get their nutrient requirements from the very shallow layer of soil
above the permafrost. The trees present have a notable climatic tolerance and survive on rather

26
Total Marks : 200
Online Prelims TEST - 7 (TEXTBOOK)
( InsightsIAS Mock Test Series for UPSC Preliminary Exam 2020 )

infertile soils.

Q Source: AR: Page 130: Fundamentals of Physical Geography: 11th NCERT

29 Consider the following statements.


Assertion (A): The share of institutional sources in the total outstanding debt of rural households
witnessed a sharp increase from the 1980s to 1990s and has continued to increase in the last two
decades.
Reason (R): The rate of growth of agricultural credit has steadily increased from the 1980s to the late
2010s.

In the context of the above, which of these is correct?


A. A is correct, and R is an appropriate explanation of A.
B. A is correct, but R is not an appropriate explanation of A.
C. A is correct, but R is incorrect.
D. Both A and R are incorrect.

Correct Answer : D

Answer Justification :

Justification: With the general deregulation of the banking sector in India post-1990s, the
following outcomes were immediately visible.

1. More than 900 rural bank branches closed down across the country.

2. The rate of growth of agricultural credit fell sharply from around 7% per annum in the 1980s
to about 2% per annum in the 1990s.

3. This retreat of public banks wreaked havoc on the rural financial market.

4. Between 1991 and 2002, the share of institutional sources in the total outstanding debt of
rural households fell from 64% to 57.1%.

5. The space vacated by institutional sources was promptly occupied by moneylenders and other
non-institutional sources.

Clearly, both A and R are wrong. We will investigate the reasons for the aforementioned problems
in more detail when we cover Ramesh Singh and 11th NCERT books.

Q Source:
https://www.insightsonindia.com/2019/08/09/insights-daily-current-affairs-pib-09-august-2019/

27
Total Marks : 200
Online Prelims TEST - 7 (TEXTBOOK)
( InsightsIAS Mock Test Series for UPSC Preliminary Exam 2020 )

30 The Annual Status of Education Report (ASER) is the largest non-governmental household survey
undertaken in rural India. It is facilitated by the

A. Barefoot College-India
B. Pratham Education Foundation
C. VIDYA
D. Make A Difference (MAD)

Correct Answer : B

Answer Justification :

Learning: 'ASER' means impact in Hindi. The survey measures the enrolment status of children
between 3-16 years and tests basic reading and arithmetic abilities of children through a detailed
process that uses a common set of testing tools and a comprehensive sampling framework.

In each rural district, a local institution carries out the survey. The entire pan-Indian process is
completed within approximately 100 days and the report is released in January each year.

The findings are disseminated widely within the government and elsewhere at the national, state,
district and village levels, even shaping their policies.

The findings of the survey have been cited in the approach paper to the 11th Planning Commission
and the Economic Survey of India.

http://www.asercentre.org/

Q Source: Syllabus: Major NGOs

31 With reference to the National Digital Health Blueprint (NDHB), consider the following statements.
1. It aims to deploy Artificial Intelligence (AI) in leveraging health records.
2. It proposes a National Digital Health Mission as a public-private partnership (PPP) with complete
functional autonomy.
3. It aims to establish national and regional registries to create single source of truth in respect of
Clinical Establishments and Healthcare Professionals.

Select the correct answer using the codes below.


A. 1 and 2 only
B. 2 and 3 only
C. 1 and 3 only
D. 1 only

Correct Answer : C

Answer Justification :

28
Total Marks : 200
Online Prelims TEST - 7 (TEXTBOOK)
( InsightsIAS Mock Test Series for UPSC Preliminary Exam 2020 )

Justification: Highlights of the National Digital Health Blueprint (NDHB):

1. It lays out the ‘building blocks’ for the implementation of the National Health Stack (NHS),
which aims to deploy Artificial Intelligence (AI) in leveraging health records.

2. Keeping true to the government’s larger agenda, of ‘data as a public good’, the blueprint
proposes the linking of multiple databases to generate greater and granular data that
can be leveraged by the public as well as private sector – including insurance
companies, hospitals, apps and researchers.

3. The blueprint proposes a National Digital Health Mission “as a purely government
organisation with complete functional autonomy adopting some features of some of the
existing National Information Utilities like UIDAI and GSTN.”

The policy document essentially lays the implementation plan and defines the ‘building blocks’ of
the NHS. In doing so, it lays down the following objectives:

1. To establish national and regional registries to create single source of truth in


respect of Clinical Establishments, Healthcare Professionals, Health Workers and Pharmacies.

2. Creating a system of Personal Health Records accessible to the citizens and to the
service providers based on citizen-consent.

3. Promoting the adoption of open standards by all the actors in the National Digital
Health Ecosystem.

4. Promoting Health Data Analytics and Medical Research.

Q Source:
https://www.insightsonindia.com/2019/08/02/insights-daily-current-affairs-pib-02-august-2019/

32 Global Coalition of the Willing on Pollinators is an initiative of

A. United Nations Environment Programme (UNEP)


B. Conference of the Parties–Convention of Biological Diversity
C. United Nations Framework Convention on Climate Change (UNFCCC)
D. Inter-governmental Panel on Climate Change (IPCC)

Correct Answer : B

29
Total Marks : 200
Online Prelims TEST - 7 (TEXTBOOK)
( InsightsIAS Mock Test Series for UPSC Preliminary Exam 2020 )

Answer Justification :

Justification: About the Global Coalition of the Willing on Pollinators:

1. The organisation was formed three years ago, to follow up on the findings of IPBES
Assessment on Pollinators, Pollination and Food Production, which found that many of the
world’s pollinator species are on the decline.

2. The initiative to form a coalition was taken by the Netherlands on December 12, 2016 at
the Conference of the Parties–Convention of Biological Diversity held in Mexico.

3. Members: The coalition now has 28 signatories including 17 European countries, five from
Latin America and the Caribbean and four from Africa.

Need:

About 16.5 per cent of vertebrate pollinators are threatened with global extinction, say
the International Union for Conservation of Nature (IUCN) Red List assessments.

The assessment highlights that 75 per cent of food crops in the world and nearly 90 per cent
of wild flowering plants depend, at least to some extent, on animal pollination.

Pollinator-dependent species include several fruits, vegetables, seeds, nuts and oil crops, which
are major sources of micronutrients, vitamins and minerals to humans.

Learning: Joining the coalition means adopting the following measures:

1. Taking action to protect pollinators and their habitats by developing and implementing
national pollinator strategies.

2. Sharing experience and lessons learnt in developing and implementing national pollinator
strategies, especially knowledge on new approaches, innovations and best practices.

3. Reaching out to seek collaboration with a broad spectrum of stakeholders—countries as well


as businesses, NGOs, farmers and local communities.

4. Developing research on pollinator conservation.

5. Supporting and collaborating with each other—and those parties that are willing to join the
coalition.

Q Source:

30
Total Marks : 200
Online Prelims TEST - 7 (TEXTBOOK)
( InsightsIAS Mock Test Series for UPSC Preliminary Exam 2020 )

https://www.insightsonindia.com/2019/08/02/insights-daily-current-affairs-pib-02-august-2019/

33 With respect to the Indian Himalayan region (IHR), consider the following statements.
1. IHR spans over more than fourteen states of India.
2. It is home to almost one-third of India’s total biodiversity.
3. IHR states share borders with six neighbouring countries.

Select the correct answer using the codes below.


A. 1 only
B. 2 and 3 only
C. 2 only
D. 1, 2 and 3

Correct Answer : B

Answer Justification :

Justification: Indian Himalayan region:

1. IHR is the section of Himalayas within India, spanning 11 Indian states (Arunachal
Pradesh, Assam, Himachal Pradesh, Jammu & Kashmir (former undivided), Manipur,
Meghalaya, Mizoram, Nagaland, Sikkim, Tripura, Uttarakhand) & 2 districts of Bengal and
that runs along 2500 km of Himalayan ranges between Indus river basin in North-
West and Brahmaputra in the East.

2. Approximately 9,000 glaciers of IHR store about 12,000 km³ of freshwater.

3. This region is endowed with rich vegetation & is home to almost 36% of India’s total
biodiversity. More than 41.5% area of IHR states is under forests, representing 1/3rd of total
forest cover of India & nearly half (47%) of the “very good” forest cover of the country.

4. The total geographical area of IHR states is approximately 591,000 sq. km (18% of
India) and it is inhabited by about 3.8% of the country’s population.

5. The strategic importance of the IHR is evident from the fact that IHR states share
borders with 6 neighbouring countries.

6. This is one of India’s major carbon sink. Besides it averts soil erosion from the world’s
youngest mountain range.

Q Source:

31
Total Marks : 200
Online Prelims TEST - 7 (TEXTBOOK)
( InsightsIAS Mock Test Series for UPSC Preliminary Exam 2020 )

https://www.insightsonindia.com/2019/08/02/insights-daily-current-affairs-pib-02-august-2019/

34 India finds organic farming an attractive proposition because


1. Export demand for organic products is on the rise.
2. Organic farming is less labour-intensive than conventional farming thus offering chances of
improving agricultural productivity.
3. Organic products are seen to be nutritionally superior to chemically produced foods.

Select the correct answer using the codes below.


A. 1 and 2 only
B. 2 and 3 only
C. 3 only
D. 1 and 3 only

Correct Answer : D

Answer Justification :

Justification: Statement 1: There is a huge unmet demand for organic products both in the
domestic as well as European markets. So, it generates incomes through international exports.

Statement 2: Since organic farming requires more labour input than conventional farming, India
will find organic farming an attractive proposition. So, 2 is wrong.

Statement 3: Studies across countries have shown that organically grown food has more nutritional
value than chemical farming thus providing us with healthy foods.

This is due to the absence of harmful fertilizers and weedicides in the crops.

Organic agriculture offers a means to substitute costlier agricultural inputs (such as HYV seeds,
chemical fertilisers, pesticides etc.) with locally produced organic inputs that are cheaper and
thereby generate good returns on investment.

Q Source: Organic farming is in news frequently

35 With reference to the Asia Cooperation Dialogue (ACD), consider the following statements.
1. It is an international non-governmental organization with an objective to ensure effective and
seamless communication between the state and non-state actors in Asia.
2. It aims to expand the trade and financial market within Asia and increase the bargaining power of
Asian countries in the global market.

Select the correct answer using the codes below.


A. 1 only
B. 2 only
C. Both 1 and 2
D. None of the above

32
Total Marks : 200
Online Prelims TEST - 7 (TEXTBOOK)
( InsightsIAS Mock Test Series for UPSC Preliminary Exam 2020 )

Correct Answer : B

Answer Justification :

Justification: Statement 1: The Asia Cooperation Dialogue (ACD) is an inter-governmental


organization created on 18 June 2002 to promote Asian cooperation at a continental level and to
help integrate separate regional organizations such as ASEAN, SAARC, the Gulf Cooperation
Council, Shanghai Cooperation Organisation and the Eurasian Economic Union.

It was inaugurated in June 2002 in Cha-Am, Thailand, where 18 Asian Foreign Ministers met
together for the first time. The ACD is a continent-wide forum, the first of its kind in Asia. More
specifically, the ACD aims to constitute the missing link in Asia by incorporating every Asian
country and building an Asian Community without duplicating other organizations or creating a bloc
against others.

Currently, the ACD comprises 34 countries: Bahrain, Bangladesh, Brunei Darussalam, Bhutan,
Cambodia, China, India, Indonesia, Iran, Japan, Kazakhstan, Republic of Korea, Kuwait, Kyrgyz
Republic, Lao PDR, Malaysia, Mongolia, Myanmar, Pakistan, Philippines, Oman, Qatar, Russia,
Saudi Arabia, Singapore, Sri Lanka, Tajikistan, Thailand, United Arab Emirates, Uzbekistan,
Vietnam, Afghanistan, Turkey and is awaiting the accession of Nepal as the 34th Member on March
2016 at the 14th Ministerial Meeting in Bangkok, Thailand.

Statement 2: The main objectives of the ACD are to:

Promote interdependence among Asian countries in all areas of cooperation by identifying


Asia's common strengths and opportunities which will help reduce poverty and improve the
quality of life for Asian people whilst developing a knowledge-based society within Asia and
enhancing community and people empowerment;
Expand the trade and financial market within Asia and increase the bargaining power of Asian
countries in lieu of competition and, in turn, enhance Asia's economic competitiveness in the
global market;
Serve as the missing link in Asian cooperation by building upon Asia's potentials and
strengths through supplementing and complementing existing cooperative frameworks so as
to become a viable partner for other regions;
Ultimately transform the Asian continent into an Asian Community, capable of interacting
with the rest of the world on a more equal footing and contributing more positively towards
mutual peace and prosperity.

33
Total Marks : 200
Online Prelims TEST - 7 (TEXTBOOK)
( InsightsIAS Mock Test Series for UPSC Preliminary Exam 2020 )

Q Source: http://www.acd-dialogue.org/about-acd.html

36 Which of the following states have been assigned certain special provisions in the Constitution under
Article 371?
1. Gujarat
2. Nagaland
3. Bihar
4. Assam

Select the correct answer using the codes below.


A. 2 only
B. 1, 2 and 4 only
C. 1 and 3 only
D. 1, 2, 3 and 4

Correct Answer : B

Answer Justification :

Justification: Overview:

1. Article 371, Maharashtra and Gujarat:

Governor has “special responsibility” to establish “separate development boards” for


“Vidarbha, Marathwada, and the rest of Maharashtra”, and Saurashtra and Kutch in Gujarat; ensure
“equitable allocation of funds for developmental expenditure over the said areas”, and “equitable
arrangement providing adequate facilities for technical education and vocational training, and
adequate opportunities for employment” under the state government.

34
Total Marks : 200
Online Prelims TEST - 7 (TEXTBOOK)
( InsightsIAS Mock Test Series for UPSC Preliminary Exam 2020 )

1. Article 371A (13th Amendment Act, 1962), Nagaland:

Inserted after a 16-point agreement between the Centre and the Naga People’s Convention in
1960, which led to the creation of Nagaland in 1963.

Parliament cannot legislate in matters of Naga religion or social practices, Naga customary law
and procedure, administration of civil and criminal justice involving decisions according to Naga
customary law, and ownership and transfer of land without concurrence of the state Assembly.

1. Article 371B (22nd Amendment Act, 1969), Assam:

The President may provide for the constitution and functions of a committee of the
Assembly consisting of members elected from the state’s tribal areas.

1. Article 371C (27th Amendment Act, 1971), Manipur:

The President may provide for the constitution of a committee of elected members from the
Hill areas in the Assembly, and entrust “special responsibility” to the Governor to ensure its
proper functioning.

Q Source:
https://www.insightsonindia.com/2019/08/07/insights-daily-current-affairs-pib-07-august-2019/

37 APEC Cooperation Network on Green Supply Chain is emerging as an important solution to promote
green trade and investment. With reference to Green supply chain, consider the following
statements.
1. It includes green and sustainable development notions to design the different parts of supply chain,
including purchasing, manufacturing, packaging, distributing, consuming and recycling.
2. It aims to enable related enterprises to comply with environmental legislation and standards,
improve environmental performance and raise up the effectiveness and efficiency of supply chain.

Select the correct answer using the codes below.


A. 1 only
B. 2 only
C. Both 1 and 2
D. None of the above

Correct Answer : C

Answer Justification :

Justification: APEC Cooperation Network on Green Supply Chain is an evolving cooperation


mechanism in order to coordinate the efforts of Green Supply Chain and to develop tools to promote

35
Total Marks : 200
Online Prelims TEST - 7 (TEXTBOOK)
( InsightsIAS Mock Test Series for UPSC Preliminary Exam 2020 )

green trade and investment, accordingly contribute to realization of Green Supply Chain in the Asia-
Pacific region.

Green growth has arisen as the new motive power for social and economic growth. In 2011, APEC
Leaders committed to “make green growth a reality in all of our economies.” In the meantime,
enhancing supply chains to reduce the time, cost, and uncertainty of moving goods and services
throughout the region continues to be another priority for APEC given its importance to trade and
economic growth.

APEC Leaders declared to achieve an APEC-wide 10 percent improvement in supply-chain


performance by 2015.

Green Supply Chain is pursuing economically and environmentally balanced supply chain, and
generally defined as using green and sustainable development notions to design the different parts
of supply chain, including purchasing, manufacturing, packaging, distributing, consuming and
recycling, etc.

Green Supply Chain aims to reduce the load and influence on the environment for products in each
stage of the life cycle, enable related enterprises to comply with environmental legislation and
standards, improve environmental performance, raise up the effectiveness and efficiency of supply
chain, and thus gain both economic and environmental benefits.

It will directly respond to APEC Leaders’ commitments on Green growth and supply chain
performance improvement. As proposed and endorsed in SOM1, the APEC High-Level Roundtable
on Green Development was successfully convened in Tianjin, China on May 8, 2014, with a fruitful
output of the Declaration, in which how to link economies to cooperation on Green Supply Chain
was highlighted and Tianjin’s offer in setting up a Green Supply Chain pilot center open to
interested economies was welcomed.

Considering the needs of APEC economies on enhancing cooperation on Green Supply Chain, it is
proposed that APEC establish a network among interested economies and identify a number of
demonstration pilot centers in economies to coordinate economies’ endeavor and activities to
promote Green Supply Chain, and contribute to the future possible actions on information sharing,
capacity building, demonstration activities, etc.

Q Source: http://www.apecgsc.org/pub/apecgsc_english/about/201604/t20160427_337.html

38 Consider the following statements.


1. The International Bamboo and Rattan Organisation (INBAR) was established in 1997 to develop and
promote innovative solutions to poverty and environmental sustainability using bamboo and rattan.
2. Membership of INBAR is open to member states of the United Nations and to intergovernmental
organizations.

Select the correct answer using the codes below.


A. 1 only
B. 2 only
C. Both 1 and 2
D. None of the above

36
Total Marks : 200
Online Prelims TEST - 7 (TEXTBOOK)
( InsightsIAS Mock Test Series for UPSC Preliminary Exam 2020 )

Correct Answer : C

Answer Justification :

Justification: Statement 1: INBAR is an independent intergovernmental organization established in


1997 to develop and promote innovative solutions to poverty and environmental sustainability using
bamboo and rattan.

INBAR evolved from an informal network of bamboo and rattan researchers set up in 1984 by
the International Development Research Centre (IDRC) of Canada.

Bangladesh, Canada, Indonesia, Myanmar, Nepal, Peru, the Philippines and the United
Republic of Tanzania made up INBAR's eight founding members. Since then, INBAR has
grown considerably in strength and scope away from a research-only organisation and
towards a more action-focused mandate.

Statement 2: Membership of INBAR is open to member states of the United Nations and to
intergovernmental organizations. INBAR currently has 46 Member States.

INBAR's Headquarters is in Beijing, China and it has regional offices in South Asia (New Delhi,
India), Central Africa (Yaoundé, Cameroon), East Africa (Addis Ababa, Ethiopia), West Africa
(Kumasi, Ghana) and Latin America and the Caribbean (Quito, Ecuador).

Q Source: https://en.wikipedia.org/wiki/International_Bamboo_and_Rattan_Organisation

https://en.wikipedia.org/wiki/List_of_intergovernmental_organizations#Asia

39 Old Stone Age sites have been studied with a lot of interest and form the backbone of cultural and
anthropological studies in the Indian subcontinent. Consider the following general observations about
them.
1. These Old Stone age sites are generally located near water sources.
2. Most of these Old stone age sites are concentrated in Southern India with the exception of Siwalik
hills and Bhimbetka.

Which of the above is/are correct?


A. 1 only
B. 2 only
C. Both 1 and 2
D. None

Correct Answer : A

Answer Justification :

37
Total Marks : 200
Online Prelims TEST - 7 (TEXTBOOK)
( InsightsIAS Mock Test Series for UPSC Preliminary Exam 2020 )

Justification: Statement 1: This may be because water sources were the lifeline of the Old stone
age people and thus the communities lived and hunted close to water sources.

Statement 2: This is incorrect. Several rock shelters and caves used by the Paleolithic people are
scattered across the subcontinent.

Some of the famous sites of Old Stone Age in India are:

The Soan valley and Potwar Plateau on the northwest India.


The Siwalik hills on the north India.
Bhimpetka in Madhya Pradesh.
Adamgarh hill in Narmada valley.
Kurnool in Andhra Pradesh and
Attirampakkam near Chennai.

Q Source: Revision: 11th NCERT: Introduction to Indian Arts

Q Source: http://saarc-sec.org/about-saarc

40 Factors that affect temperature at a particular region include


1. Circulation of planetary and local winds
2. Altitude and terrain of the place
3. Distance of the region from poles or equator
4. Movement of Ocean currents

Select the correct answer using the codes below.


A. 2 and 3 only
B. 1, 2 and 3 only
C. 1 and 4 only
D. 1, 2, 3 and 4

Correct Answer : D

Answer Justification :

Learning: Ocean Waves do not induce movement of water in the ocean from one place to another.
Movement of water is done by ocean currents.

If water is not moved from one place to the other, there is no circulation of temperature, nutrients,
water density or salinity.

So, ocean waves affect the temperature distribution the least. Ocean ‘currents’ do. Ocean currents,
warm or cold, carry heat/coolness from one sea region to the other and affect the temperature
distribution.

Q Source: Page 82: Fundamentals of Physical Geography: 11th NCERT

38
Total Marks : 200
Online Prelims TEST - 7 (TEXTBOOK)
( InsightsIAS Mock Test Series for UPSC Preliminary Exam 2020 )

41 Which of these is/are international NGOs, or their branches, working in India?


1. HelpAge India
2. Save the Children
3. Oxfam

Select the correct answer using the codes below.


A. 3 only
B. 2 and 3 only
C. 1, 2 and 3
D. 1 and 2 only

Correct Answer : C

Answer Justification :

Justification: Statement 1: These are some of the major NGOs working in India.

Save the Children Bal Raksha Bharat.


Oxfam.
HelpAge India.
CARE India.
World Vision India.
Plan India.
Action Aid.
Greenpeace India.
Oxfam India’s vision is to help create an equal, just and sustainable society by empowering
the underprivileged. Oxfam India believes in the ‘Right to Life with Dignity for All’. This is
fulfilled by engaging empowered citizens to become active and supportive partners,
advocating an effective and accountable State and making markets work for the
underprivileged.
Another important NGO - Save the Children started working as an independent entity in India
— Save the Children Bal Raksha Bharat in April, 2008. Since past eight years, the
organization has consolidated its place as the most respectable organization working to
protect the rights of children in the most marginalized areas of the country.
HelpAge India is a leading charity in India working with and for disadvantaged elderly for
nearly 4 decades. It was set up in 1978 and is registered under the Societies’ Registration Act
of 1860. There are an estimated 106 million elderly in India. HelpAge advocates for their
needs such as for Universal Pension, quality healthcare, action against Elder Abuse and many
more at a national, state and societal level with Central and State governments. It advocates
for elder friendly policies and their implementation thereof. It works hand-in-hand with Senior
Citizens Associations understanding elder needs working with and for them.

Q Source: Syllabus: Important NGOs

42 “Quarterly Statistics on Deposits & Credit of Scheduled Commercial Banks in India” report is
published by

39
Total Marks : 200
Online Prelims TEST - 7 (TEXTBOOK)
( InsightsIAS Mock Test Series for UPSC Preliminary Exam 2020 )

A. Reserve Bank of India


B. NABARD
C. Ministry of Finance
D. Ministry of Economic Affairs

Correct Answer : A

Answer Justification :

Justification: You can access the report here


https://www.rbi.org.in/Scripts/QuarterlyPublications.aspx?head=Quarterly%20Statistics%20on%20
Deposits%20and%20Credit%20of%20Scheduled%20Commercial%20Banks

Other quarterly reports published by RBI include –

Macroeconomic and Monetary Developments


Variation to Foreign Exchange Reserves in India: Sources, Arbitrage and Costs
Quarterly Industrial Outlook Survey
Consumer Confidence Survey
Inflation Expectations Survey of Households

Q Source: https://www.rbi.org.in/Scripts/Publications.aspx?publication=Quarterly

43 What is/are the difference(s) between extrusive and intrusive rocks?


1. Extrusive rocks are fine grained, whereas intrusive rocks are coarse-grained.
2. Extrusive rocks form over a much longer duration of time compared to intrusive rocks.

Select the correct answer using the codes below.


A. 1 only
B. 2 only
C. Both 1 and 2
D. None of the above

Correct Answer : A

Answer Justification :

Concept: The key difference between intrusive and extrusive rocks is that the intrusive rocks are
formed from magma whereas the extrusive rocks are formed from lava.

Rest of the differences follow from this basic structure.

Intrusive rocks: With no air to cool the magma, these rocks are formed very slowly. Composition
of these rocks reflects presence of large crystals. These crystals interlock to form the rock.

These rocks take a very large amount of time to solidify and they remain buried deep inside

40
Total Marks : 200
Online Prelims TEST - 7 (TEXTBOOK)
( InsightsIAS Mock Test Series for UPSC Preliminary Exam 2020 )

the surface of the earth being surrounded by country rocks that have been there already.
Very slow cooling means that these rocks remain coarse-grained.
Some of the perfect examples of intrusive rocks are the diorite, gabbro, and granite.
Much of the core of various mountain ranges around the world is made up of these intrusive
rocks.

Extrusive Rocks: Sometimes, molten rocks find a way to come out of the surface of the earth
through cracks and openings.

This magma flows in the form of lava and cools down quickly as it comes into contact with air.
Igneous rocks that are formed from the magma that pours out of the surface of the earth are
called extrusive rocks.
As these rocks cool down and solidify very quickly, they do not get sufficient time to form
large crystals. Thus, they have small crystals and boast a fine texture.

Q Source: Additional Research: Glossary: Fundamentals of Physical Geography: 11th NCERT

44 If a river in India is to be converted into National Waterways (NWs), it can be done via a/an

A. Central Executive fiat


B. Legislation
C. Constitutional amendment only
D. Order of the State Government

Correct Answer : B

Answer Justification :

Learning: India has huge natural geographical potential for inland waterways as 14,500 km of river
channels are navigable.

Development inland waterways can spur industrial growth and tourism potential along the
waterway especially in plain areas of North India.
The statutory status to the inland waterways would pave for investments in inland waterways
transportation sector which has been neglected in the past.
Parliament had enacted a legislation allowing 106 rivers across the country to be converted
into National Waterways (NWs) in addition to the existing 5 such NWs.
The law was enacted in a bid to boost movement of goods and passengers via rivers and
reduce transportation costs substantially.

Q Source: Previous months Current Affairs

45 Which of the following is/are harmful environmental effects of nitrogen oxides?


1. It causes irritant red haze in the traffic and congested places.
2. It can damage plant leaves and retard the rate of photosynthesis.

41
Total Marks : 200
Online Prelims TEST - 7 (TEXTBOOK)
( InsightsIAS Mock Test Series for UPSC Preliminary Exam 2020 )

Which of the above is/are correct?


A. 1 only
B. 2 only
C. Both 1 and 2
D. None

Correct Answer : C

Answer Justification :

Justification: Dinitrogen and dioxygen are the main constituents of air.

These gases do not react with each other at a normal temperature. At high altitudes when
lightning strikes, they combine to form oxides of nitrogen.

NO2 is oxidised to nitrate ion, which is washed into soil, where it serves as a fertilizer. In an
automobile engine, (at high temperature) when fossil fuel is burnt, dinitrogen and dioxygen
combine to yield significant quantities of nitric oxide.

The irritant red haze in the traffic and congested places is due to oxides of nitrogen. Higher
concentrations of NO2 damage the leaves of plants and retard the rate of photosynthesis.
Nitrogen dioxide is a lung irritant that can lead to an acute respiratory disease in children.

It is toxic to living tissues and also harmful to various textile fibres and metals.

Q Source: Page 409: Chemistry Unit 14: XIth NCERT

46 Abnormal cooling and accumulation of cold water occurring in the central and eastern Pacific Ocean
and the strengthening of Tropical pacific trade winds is an unusual occurrence associated with

A. El Nino
B. Indian Ocean Dipole
C. La Niña
D. Either (a) or (b)

Correct Answer : C

Answer Justification :

Justification & Learning: La Niña refers to the extensive cooling of the central and eastern
tropical Pacific Ocean, often accompanied by warmer than normal sea surface temperatures (SSTs)
in the western Pacific, and to the north of Australia.

42
Total Marks : 200
Online Prelims TEST - 7 (TEXTBOOK)
( InsightsIAS Mock Test Series for UPSC Preliminary Exam 2020 )

Changes to the atmosphere and ocean circulation during La Niña events include:

Sustained cooler-than-usual SSTs across the central and eastern tropical Pacific Ocean.
Increased convection or cloudiness over tropical Australia, Papua New-Guinea, and Indonesia.
An increase in strength of the trade winds (easterlies) across the tropical Pacific Ocean (but
not necessarily in the Australian region).

Q Source: Revision: Glossary: Fundamentals of Physical Geography: 11th NCERT

47 Agricultural Lime is usually added to soil in order to


1. Increase the oil PH
2. Improve crop yields
3. Provide a source of calcium and magnesium for plants
4. Allow improved water penetration for acidic soils

Which of the above is/are correct?


A. 1, 2, 3 and 4
B. 1 only
C. 1 and 3 only
D. 2 and 4 only

Correct Answer : A

Answer Justification :

Justification: Agricultural lime, also called aglime, agricultural limestone, garden lime or liming, is
a soil additive made from pulverized limestone or chalk.

The primary active component is calcium carbonate. Additional chemicals vary depending on the
mineral source and may include calcium oxide. Unlike the types of lime called quicklime (calcium
oxide) and slaked lime (calcium hydroxide), powdered limestone does not require lime burning in a
lime kiln; it only requires milling.

The effects of agricultural lime on soil are:

it increases the pH of acidic soil (the lower the pH the more acidic the soil); in other words,
soil acidity is reduced and alkalinity increased
it provides a source of calcium and magnesium for plants
it permits improved water penetration for acidic soils
it improves the uptake of major plant nutrients (nitrogen, phosphorus, and potassium) of
plants growing on acid soils

Q Source: AR: Page 403: Chemistry Unit 14: XIth NCERT

48 Photochemical smog generally forms in

43
Total Marks : 200
Online Prelims TEST - 7 (TEXTBOOK)
( InsightsIAS Mock Test Series for UPSC Preliminary Exam 2020 )

A. Low lying mountain valleys with little or no pollution


B. Cold and moist conditions
C. Polluted urban warm, dry and sunny climate
D. Regions where icefall is predominant

Correct Answer : C

Answer Justification :

Justification: Photochemical smog occurs in warm, dry and sunny climate. The main components
of the photochemical smog result from the action of sunlight on unsaturated hydrocarbons and
nitrogen oxides produced by automobiles and factories.

Photochemical smog has high concentration of oxidising agents and is, therefore, called as oxidising
smog.

Learning: The common components of photochemical smog are ozone, nitric oxide, acrolein,
formaldehyde and peroxyacetyl nitrate (PAN).

Photochemical smog causes serious health problems. Both ozone and PAN act as powerful eye
irritants.

Ozone and nitric oxide irritate the nose and throat and their high concentration causes headache,
chest pain, dryness of the throat, cough and difficulty in breathing.

Photochemical smog leads to cracking of rubber and extensive damage to plant life. It also causes
corrosion of metals, stones, building materials, rubber and painted surfaces.

Q Source: Page 403: Chemistry Unit 14: XIth NCERT

49 Which of these nations lie both in Northern and Southern Hemisphere?


1. Brazil
2. Maldives
3. Thailand
4. Morocco

Select the correct answer using the codes below.


A. 1 and 2 only
B. 3 and 4 only
C. 1, 2 and 3 only
D. 2, 3 and 4 only

Correct Answer : A

Answer Justification :

44
Total Marks : 200
Online Prelims TEST - 7 (TEXTBOOK)
( InsightsIAS Mock Test Series for UPSC Preliminary Exam 2020 )

Learning:

The image shows equator passing through countries, these are the ones that lie in both
hemispheres.

Q Source: Map-based: World map

50 Which of the following chemicals is NOT associated with the depletion of Ozone layer?

A. Nitrogen dioxide
B. Methane
C. Chlorine monoxide
D. Potassium hydroxide

Correct Answer : D

Answer Justification :

Learning: In 1980s atmospheric scientists working in Antarctica reported about depletion of ozone
layer commonly known as ozone hole over the South Pole.

It was found that a unique set of conditions was responsible for the ozone hole.

In summer season, nitrogen dioxide and methane react with chlorine monoxide and chlorine atoms
forming chlorine sinks, preventing much ozone depletion, whereas in winter, special type of clouds

45
Total Marks : 200
Online Prelims TEST - 7 (TEXTBOOK)
( InsightsIAS Mock Test Series for UPSC Preliminary Exam 2020 )

called polar stratospheric clouds are formed over Antarctica.

These polar stratospheric clouds provide surface on which chlorine nitrate formed gets hydrolysed
to form hypochlorous acid. It also reacts with hydrogen chloride produced as per reaction to give
molecular chlorine.

The chlorine radicals formed after some other reactions, initiate the chain reaction for ozone
depletion.

Q Source: Page 405: Chemistry Unit 14: XIth NCERT

51 Consider the following statements.


The English Channel
1. is a part of the Arctic Ocean
2. separates the island of Britain from northern France
3. joins the North Sea to the Atlantic Ocean

Select the correct answer using the codes below.


A. 1, 2 and 3
B. 1 and 2 only
C. 2 and 3 only
D. 3 only

Correct Answer : C

Answer Justification :

Justification: French inventor crosses the English Channel on his hoverboard.

Key facts:

The English Channel is a part of the Atlantic Ocean.

It separates the island of Britain (part of the UK) from northern France and joins
the North Sea to the Atlantic Ocean.

It’s approximately 350 miles long, and at its narrowest in the Strait of Dover.

46
Total Marks : 200
Online Prelims TEST - 7 (TEXTBOOK)
( InsightsIAS Mock Test Series for UPSC Preliminary Exam 2020 )

Q Source:
https://www.insightsonindia.com/2019/08/05/insights-daily-current-affairs-pib-05-august-2019/

52 Consider the following about Polychlorinated biphenyl (PCB).


1. Polychlorinated biphenyls were once widely deployed as dielectric and coolant fluids in electrical
apparatus
2. These are classified as a persistent organic pollutant and banned under the Stockholm Convention.

Select the correct answer using the codes below.


A. 1 only
B. 2 only
C. Both 1 and 2
D. None of the above

Correct Answer : C

Answer Justification :

Justification: Polychlorinated biphenyls were once widely deployed as dielectric and coolant fluids
in electrical apparatus, carbonless copy paper and in heat transfer fluids. Because of their
longevity, PCBs are still widely in use, even though their manufacture has declined drastically since
the 1960s, when a host of problems were identified

47
Total Marks : 200
Online Prelims TEST - 7 (TEXTBOOK)
( InsightsIAS Mock Test Series for UPSC Preliminary Exam 2020 )

PCB production was banned by the United States Congress in 1979 and by the Stockholm
Convention on Persistent Organic Pollutants in 2001.

Generally, PCBs are not very soluble in water, but readily soluble in fats. This solubility in fat
explains why PCBs can build up in animal fat and along the food chain.

But, typically, PCBs are either broken down in the environment by sunlight or by microorganisms.
Sunlight plays an important role in the breakdown of PCBs when they are in the air, shallow water,
or surface soils. Microorganisms, such as bacteria, algae, or fungi, biodegrade PCBs when found in
soil or sediments. So, 2 is partly incorrect.

Learning: According to the U.S. Environmental Protection Agency (EPA), PCBs cause cancer in
animals and are probable human carcinogens.

Typically, PCBs are either broken down in the environment by sunlight or by microorganisms.
Sunlight plays an important role in the breakdown of PCBs when they are in the air, shallow water,
or surface soils. Microorganisms, such as bacteria, algae, or fungi, biodegrade PCBs when found in
soil or sediments.

Many rivers and buildings including schools, parks, and other sites are contaminated with PCBs,
and there have been contaminations of food supplies with the toxins. So, they have a much wider
area of influence.

The bromine analogues of PCBs are polybrominated biphenyls (PBBs), which have analogous
applications and environmental concerns.

Q Source: AR: Page 407: Chemistry Unit 14: XIth NCERT

53 In which of the following trophic levels, you are likely to find the highest concentration of an organic
toxin that has been found widely in ocean water and the aquatic animals inhabiting it?

A. Phytoplankton
B. Aquatic plants
C. Small fishes at lower trophic levels
D. Human being who consumes sea food

Correct Answer : D

Answer Justification :

Learning: Most of the organic toxins are water insoluble and non-biodegradable.

These high persistent toxins are, therefore, transferred from lower trophic level to higher trophic
level through food chain (shown below).

Over time, the concentration of toxins in higher animals reaches a level which causes serious
metabolic and physiological disorders.

48
Total Marks : 200
Online Prelims TEST - 7 (TEXTBOOK)
( InsightsIAS Mock Test Series for UPSC Preliminary Exam 2020 )

Q Source: Page 409: Chemistry Unit 14: XIth NCERT

54 India has signed the Singapore Convention on Mediation. What will be the positive implications of
this move?
1. It will allow Indian individuals and entities to move arbitration and mediation cases from any law
court in India to the UN Committee on Mediation.
2. It will give a positive signal to foreign investors about India’s commitment to adhere to international
practice on Alternative Dispute Resolution (ADR).

Select the correct answer using the codes below.


A. 1 only
B. 2 only
C. Both 1 and 2
D. None of the above

Correct Answer : B

Answer Justification :

Justification: India has signed the United Nations Convention on International Settlement
Agreements (UNISA).

About UNISA:

49
Total Marks : 200
Online Prelims TEST - 7 (TEXTBOOK)
( InsightsIAS Mock Test Series for UPSC Preliminary Exam 2020 )

The United Nations General Assembly adopted the United Nations Convention on International
Settlement Agreements Resulting from Mediation (“the Convention”) on 20th December 2018.

The convention is also known as the “Singapore Convention on Mediation” (the Convention).

Key features:

1. The Convention provides a uniform and efficient framework for the enforcement of
international settlement agreements resulting from mediation and for allowing parties to
invoke such agreements, akin to the framework that the Convention on the Recognition and
Enforcement of Foreign Arbitral Awards (New York, 1958) (the “New York Convention”)
provides for arbitral awards.

2. The Convention defines two additional grounds upon which a court may, on its own motion,
refuse to grant relief. Those grounds relate to the fact that a dispute would not be capable of
settlement by mediation or would be contrary to public policy.

Benefit:

Signing of the Convention will boost the confidence of the investors and shall provide a positive
signal to foreign investors about India’s commitment to adhere to international practice
on Alternative Dispute Resolution (ADR).

Q Source:
https://www.insightsonindia.com/2019/08/08/insights-daily-current-affairs-pib-08-august-2019/

55 Consider the following statements.


1. SANKALP aims to implement the mandate of the National Skill Development Mission (NSDM).
2. SANKALP marks a shift in government’s implementation strategy in the field of vocational
education moving from practical training to theoretical training emphasizing on college education.

Select the correct answer using the codes below.


A. 1 only
B. 2 only
C. Both 1 and 2
D. None of the above

Correct Answer : A

Answer Justification :

Justification: SANKALP is a Centrally sponsored scheme of Ministry of Skill Development &


Entrepreneurship (MSDE). It is an outcome focused scheme marking shift in government’s
implementation strategy in vocational education and training from inputs to results. SANKALP aims
to implement the mandate of the National Skill Development Mission (NSDM).

50
Total Marks : 200
Online Prelims TEST - 7 (TEXTBOOK)
( InsightsIAS Mock Test Series for UPSC Preliminary Exam 2020 )

SANKALP will provide market relevant training to 3.5 crore youth.

Objective: The Objective of the project is to enhance institutional mechanisms for skills
development and increase access to quality and market-relevant training for the work force.

What it does? The scheme provides the required impetus to the National Skill Development
Mission, 2015 and its various sub missions. It is aligned to flagship Government of India programs
such as Make in India and Swachhta Abhiyan and aims at developing globally competitive workforce
for domestic and overseas requirements.

The Key result areas for the project include Institutional Strengthening at the National and State
Levels for Planning, Delivering, and Monitoring High-Quality Market-Relevant Training; Improved
Quality and Market Relevance of Skills Development Programs; Improved access to and completion
of skills training for female trainees and other disadvantaged groups; and Expanding skills training
through private-public partnerships (PPPs).

Q Source: Frequently in news

56 Consider the following statements.


1. C4 plants are corn, sorghum, sugarcane and millet.
2. C4 photosynthetic system maximises the availability of CO2 while minimising water loss.

Which of the above is/are correct?


A. 1 only
B. 2 only
C. Both 1 and 2
D. None of the above

Correct Answer : C

Answer Justification :

Justification: The majority of plants and crop plants are C3 plants, referring to the fact that the
first carbon compound produced during photosynthesis contains three carbon atoms. Under high
temperature and light, however, oxygen has a high affinity for the photosynthetic enzyme Rubisco.

Oxygen can bind to Rubisco instead of carbon dioxide, and through a process called
photorespiration, oxygen reduces C3 plant photosynthetic efficiency and water use efficiency. In
environments with high temperature and light, that tend to have soil moisture limitations, some
plants evolved C4 photosynthesis. A unique leaf anatomy and biochemistry enables C4 plants to
bind carbon dioxide when it enters the leaf and produces a 4-carbon compound that transfers and
concentrates carbon dioxide in specific cells around the Rubisco enzyme, significantly improving the
plant's photosynthetic and water use efficiency.

As a result in high light and temperature environments, C4 plants tend to be more productive than
C3 plants. Examples of C4 plants include corn, sorghum, sugarcane, millet, and switchgrass.
However, the C4 anatomical and biochemical adaptations require additional plant energy and

51
Total Marks : 200
Online Prelims TEST - 7 (TEXTBOOK)
( InsightsIAS Mock Test Series for UPSC Preliminary Exam 2020 )

resources than C3 photosynthesis, and so in cooler environments, C3 plants are typically more
photosynthetically efficient and productive.

To summarize, C3 plants are numerous and they use the C3 pathway. C4 plants mostly dwell in
warm water and moist climate. C4 cycle is the alternate pathway of calvin cycle which takes place
during the dark phase of photosynthesis.

The evolution of the C4 photosynthetic system is probably one of the strategies for maximising the
availability of CO2 while minimising water loss.

C4 plants are twice as efficient as C3 plants in terms of fixing carbon (making sugar). However, a
C4 plant loses only half as much water as a C3 plant for the same amount of CO2 fixed.

Q Source: AR: Page 189: Biology: XIth NCERT

57 Which of the following plant micronutrients is associated with chlorophyll formation in plants?

A. Magnesium
B. Iron
C. Copper
D. All of the above

Correct Answer : D

Answer Justification :

Justification & Learning: Magnesium forms a part of the chlorophyll molecule - essential in
photosynthesis.

Iron is taken up in ferrous and ferric forms by plants. It acts as a catalyst in the production of
chlorophyll.

Cu+2 (copper) can be absorbed through leaves. It becomes very toxic if too much applied. It is also
a catalyst in chlorophyll formation.

So, all given options are correct.

Q Source: Improvisation: Chapter 12: Biology: XIth NCERT

58 Which of the following are nitrogen-fixing bacteria?


1. Rhodospirillum
2. Azotobacter
3. Anabaena
4. Dibosenfeca

Select the correct answer using the codes below.


A. 1, 2 and 3 only

52
Total Marks : 200
Online Prelims TEST - 7 (TEXTBOOK)
( InsightsIAS Mock Test Series for UPSC Preliminary Exam 2020 )

B. 2 and 4 only
C. 1 and 3 only
D. 1, 2, 3 and 4

Correct Answer : A

Answer Justification :

Justification: The nitrogen-fixing microbes could be free-living or symbiotic. Examples of free-


living nitrogen-fixing aerobic microbes are Azotobacter and Beijernickia while Rhodospirillum is
anaerobic and Bacillus free-living.

In addition, a number of cyanobacteria such as Anabaena and Nostoc are also free-living nitrogen-
fixers.

Several types of symbiotic biological nitrogen fixing associations are known. The most prominent
among them is the legume-bacteria relationship.

Both Rhizobium and Frankia are free-living in soil, but as symbionts, can fix atmospheric nitrogen.

Q Source: Page 202: Biology: XIth NCERT

59 In a hyperloop transportation system, as proposed by Tesla earlier


1. A pod-like vehicle is propelled through a near-vacuum tube at speeds matching that of a typical
aircraft
2. The cost of construction has been estimated to be double that of high speed trains per kilometre of
track

Select the correct answer using the codes below.


A. 1 only
B. 2 only
C. Both 1 and 2
D. None of the above

Correct Answer : A

Answer Justification :

Justification: Richard Branson’s Virgin Hyperloop One has signed an agreement with the
Maharashtra government to build a Hyperloop that will cut the travel time between Mumbai and
Pune to 25 minutes from the 3 hours it now takes by road.

What is hyperloop transportation system?

It is a transportation system where a pod-like vehicle is propelled through a near-vacuum tube


connecting cities at speeds matching that of an aircraft.

53
Total Marks : 200
Online Prelims TEST - 7 (TEXTBOOK)
( InsightsIAS Mock Test Series for UPSC Preliminary Exam 2020 )

The hyperloop concept is a brainchild of Tesla founder Elon Musk. US-based Hyperloop Transport
Technology (HTT) claimed it costs $40 million per kilometre to build a hyperloop system while
building a high-speed train line would cost almost twice. The hyperloop system is being designed to
transport passengers and freight.

How it operates?

1. In hyperloop transportation, custom-designed capsules or pods are expected to zip smoothly


through continuous steel tubes which are held at partial vacuum. The pod which sandwiches
the passenger compartment between an air compressor upfront and a battery compartment in
the rear is supported by air caster skis at the bottom.

2. The skis float on a thin layer of air provided under high pressure, eliminating rolling
resistance and allowing for movement of the pods at high speeds. These capsules are
expected to be driverless with estimated speeds of 1,000 km/h. Linear induction motors that
are placed along the tube control the speed of the pod. Electronically-assisted acceleration
and braking determines the speed of the capsule.

Q Source:
https://www.insightsonindia.com/2019/08/03/insights-daily-current-affairs-pib-03-august-2019/

60 Which of the following roles is/are played by Abscisic acid (ABA) in plant development?
1. It is a plant growth hormone.
2. It increases the tolerance of plants to various kinds of stresses.

Which of the above is/are correct?


A. 1 only
B. 2 only
C. Both 1 and 2
D. None

Correct Answer : C

Answer Justification :

Justification: Statement 1 and 2: Abscisic acid (ABA) was discovered for its role in regulating
abscission and dormancy. But like other PGRs, it also has other wide ranging effects on plant
growth and development.

It acts as a general plant growth inhibitor and an inhibitor of plant metabolism. ABA inhibits seed
germination. ABA stimulates the closure of stomata in the epidermis and increases the tolerance of
plants to various kinds of stresses. Therefore, it is also called the stress hormone.

ABA plays an important role in seed development, maturation and dormancy. By inducing
dormancy, ABA helps seeds to withstand desiccation and other factors unfavourable for growth

54
Total Marks : 200
Online Prelims TEST - 7 (TEXTBOOK)
( InsightsIAS Mock Test Series for UPSC Preliminary Exam 2020 )

Q Source: Chapter 15: Biology: XIth NCERT

61 Photoperiodism is a phenomenon related to


1. Flowering in plants
2. Mode of photosynthesis in plants

Which of the above is/are correct?


A. 1 only
B. 2 only
C. Both 1 and 2
D. None of the above

Correct Answer : A

Answer Justification :

Learning: Photoperiodism is the physiological reaction of organisms to the length of day or night.
It occurs in plants and animals.

It has been observed that some plants require a periodic exposure to light to induce
flowering. It is also seen that such plants are able to measure the duration of exposure to
light.
For example, some plants require the exposure to light for a period exceeding a well defined
critical duration, while others must be exposed to light for a period less than this critical
duration before the flowering is initiated in them.
Together with temperature changes, photoperiod provokes changes in the colour of fur and
feathers, migration, entry into hibernation, sexual behaviour etc. For e.g. the singing
frequency of birds such as the canary depends on the photoperiod.

Q Source: Page 251: Biology 11th NCERT

62 Consider the following statements.


1. The expansion created by the “Big Bang” continues even to the present day.
2. Soon after the Big Bang, the Universe became highly opaque and temperatures started rising till
the atmosphere was formed.

Which of the above is/are correct?


A. 1 only
B. 2 only
C. Both 1 and 2
D. None of the above

Correct Answer : A

Answer Justification :

55
Total Marks : 200
Online Prelims TEST - 7 (TEXTBOOK)
( InsightsIAS Mock Test Series for UPSC Preliminary Exam 2020 )

Justification: In the beginning, all matter forming the universe existed in one place in the form of a
“tiny ball” (singular atom) with an unimaginably small volume, infinite temperature and infinite
density.

At the Big Bang the “tiny ball” exploded violently. This led to a huge expansion. It is now
generally accepted that the event of big bang took place 13.7 billion years before the present.
The expansion continues even to the present day. As it grew, some energy was converted into
matter
Within 300,000 years from the Big Bang, temperature dropped to 4,500 K and gave rise to
atomic matter. The universe became transparent. The actual reason for expansion of Universe
has not been fully established by the science community even till date.

Q Source: Page 14: Chapter 2: Fundamentals of Physical Geography: 11th NCERT

63 The Terrestrial and Jovian planets are largely differentiated on the basis of loss of atmosphere from
the former. Which of the following characteristics of terrestrial planets enabled this loss of
atmosphere?
1. Their size were bigger compared to the Jovian planets
2. Their proximity to Sun

Which of the above is/are correct?


A. 1 only
B. 2 only
C. Both 1 and 2
D. None

Correct Answer : B

Answer Justification :

Justification: Statement 1: It can be easily discarded. Jovian planets (Jupiter, Saturn etc) are
bigger in size than terrestrial planets (earth, Venus etc).

Statement 2: Gravity of a planet depends on its mass and size. Since the terrestrial planets were
small, gravity force was less and it could not keep the gases in tact with the planet. Hence, gases
escaped causing loss of atmosphere.

The terrestrial planets were formed in the close vicinity of the parent star where it was too warm
for gases to condense to solid particles. Moreover, the solar wind was most intense nearer the sun;
so, it blew off lots of gas and dust from the terrestrial planets causing loss of atmosphere.

Q Source: Page 16: Chapter 2: Fundamentals of Physical Geography: 11th NCERT

64 Which of the following causes “Sea floor spreading”?

A. Weight of the ocean bed

56
Total Marks : 200
Online Prelims TEST - 7 (TEXTBOOK)
( InsightsIAS Mock Test Series for UPSC Preliminary Exam 2020 )

B. Due to gravitational force


C. Constant volcanic eruptions at the ocean floor
D. Presence of Seabed methane

Correct Answer : C

Answer Justification :

Justification: Constant eruptions at the crest of oceanic ridges cause the rupture of the oceanic
crust and the new lava wedges into it, pushing the oceanic crust on either side. The ocean floor,
thus spreads, known as sea floor spreading.

Sea floor spreading is verified using these phenomena:

It was realised that all along the mid-oceanic ridges, volcanic eruptions are common and they
bring huge amounts of lava to the surface in this area. The sediments on the ocean floor are
unexpectedly very thin.
The age of the rocks increases as one moves away from the crest.
The ocean crust rocks are much younger than the continental rocks.
The sediments on the ocean floor are unexpectedly very thin.
The deep trenches have deep-seated earthquake occurrences while in the mid-oceanic ridge
areas, the quake foci have shallow depths. It means lava is in close vicinity.

Q Source: Page 33: Fundamentals of Physical Geography: 11th NCERT

65 With reference to the provisions under the PM Kisan Maan Dhan Yojana, consider the following
statements.
1. The scheme is voluntary and contributory for farmers in the entry age group of 18 to 40 years.
2. A monthly pension of Rs. 3000/– will be provided to them on attaining the age of 60 years.
3. The spouse is also eligible to get a separate pension of Rs.3000/- upon making separate
contributions to the Fund.
4. The Life Insurance Corporation of India (LIC) shall be the Pension Fund Manager and responsible
for Pension pay out.

Select the correct answer using the codes below.


A. 1 and 2 only
B. 2, 3 and 4 only
C. 1 and 4 only
D. 1, 2, 3 and 4

Correct Answer : D

Answer Justification :

Justification: Registration opens for PM Kisan Maan Dhan Yojana.

57
Total Marks : 200
Online Prelims TEST - 7 (TEXTBOOK)
( InsightsIAS Mock Test Series for UPSC Preliminary Exam 2020 )

Aim: To improve the life of small and marginal farmers of the country.

Salient features of the scheme:

1. The scheme is voluntary and contributory for farmers in the entry age group of 18 to
40 years.

2. A monthly pension of Rs. 3000/– will be provided to them on attaining the age of 60 years.

3. The farmers will have to make a monthly contribution of Rs.55 to Rs.200, depending
on their age of entry, in the Pension Fund till they reach the retirement date i.e. the age of 60
years.

4. The Central Government will also make an equal contribution of the same amount in
the pension fund.

5. The spouse is also eligible to get a separate pension of Rs.3000/- upon making separate
contributions to the Fund.

6. The Life Insurance Corporation of India (LIC) shall be the Pension Fund Manager and
responsible for Pension pay out.

7. In case of death of the farmer before retirement date, the spouse may continue in the
scheme by paying the remaining contributions till the remaining age of the deceased farmer.

8. If the spouse does not wish to continue, the total contribution made by the farmer along
with interest will be paid to the spouse.

9. If there is no spouse, then total contribution along with interest will be paid to the
nominee.

10. If the farmer dies after the retirement date, the spouse will receive 50% of the pension as
Family Pension.

11. After the death of both the farmer and the spouse, the accumulated corpus shall be
credited back to the Pension Fund.

12. The beneficiaries may opt voluntarily to exit the Scheme after a minimum period of 5
years of regular contributions.

58
Total Marks : 200
Online Prelims TEST - 7 (TEXTBOOK)
( InsightsIAS Mock Test Series for UPSC Preliminary Exam 2020 )

13. On exit, their entire contribution shall be returned by LIC with an interest equivalent to
prevailing saving bank rates.

14. The farmers, who are also beneficiaries of PM-Kisan Scheme, will have the option to
allow their contribution debited from the benefit of that Scheme directly.

15. In case of default in making regular contributions, the beneficiaries are allowed
to regularize the contributions by paying the outstanding dues along with prescribed
interest.

Q Source:
https://www.insightsonindia.com/2019/08/10/insights-daily-current-affairs-pib-10-august-2019/

66 What could be a reason that the regions near Alaska, Canada, and northern Europe consistently
experience “high ranges of tides”?
1. Respective latitudes
2. Constriction of the oceans due to landmass distribution in the Northern hemisphere

Select the correct answer using the codes below.


A. 1 only
B. 2 only
C. Both 1 and 2
D. None of the above

Correct Answer : B

Answer Justification :

Justification: Statement 1: That many of the areas of the world with high ranges of tides are in the
areas of Alaska, Canada, and northern Europe has created a misconception that the range of tide
increases with increasing latitude (as one moves farther from the equator and closer to the poles).
This is incorrect.

Statement 2: Increased tidal ranges in these areas are created by the positions and configurations
of the continents in the northern hemisphere. In the higher latitudes of the northern hemisphere,
the continents of North America, Europe, and Asia are pressed closer together. This “constriction”
of the oceans creates the effect of a higher range of tides.

In the higher latitudes of the southern hemisphere, in the southern tips of South America, southern
Africa, Australia, and Antarctica, tidal ranges are not increased. In these areas the continents are
not pressed closely together, there is not a “constriction” of the oceans, and the tidal ranges are not
increased.

Q Source: AR: Page 122: Fundamentals of Physical Geography: 11th NCERT

59
Total Marks : 200
Online Prelims TEST - 7 (TEXTBOOK)
( InsightsIAS Mock Test Series for UPSC Preliminary Exam 2020 )

67 What is unique about the Equatorial counter-currents?


1. They are the only current to flow between two eastward-flowing ocean currents.
2. They flow in a direction opposite to that of the surface winds
3. They circulate from equator the poles uninterrupted.

Which of the above is/are correct?


A. 1 only
B. 2 and 3 only
C. 2 only
D. 1 and 3 only

Correct Answer : C

Answer Justification :

Justification: Equatorial counter-currents are major surface flows that carry water eastward in the
Atlantic, Indian, and Pacific Oceans.

They are located near the equator and are sandwiched between two westward-flowing
currents, the North Equatorial Current and the South Equatorial Current. So, option D is
incorrect.

Equatorial counter-currents are unique, in that they flow in the opposite direction of the
surface winds. The other major surface currents in the tropics flow in the same direction as
the prevailing winds.

The equatorial counter-currents are thus driven by a distinct surface wind pattern in the
tropics. Strong westward trade winds result in westward surface flow in most of the tropical
Atlantic and Pacific Oceans.

However, several hundred miles north of the equator the winds are much weaker, in
comparison. The stronger winds to the south pile up water where the winds are weak.

As a result, the surface of the ocean can be up to 6 inches higher. The excess water flows
eastward under the influence of the Earth’s rotation, giving rise to the equatorial counter-
currents.

60
Total Marks : 200
Online Prelims TEST - 7 (TEXTBOOK)
( InsightsIAS Mock Test Series for UPSC Preliminary Exam 2020 )

Q Source: AR: Fig 14.3: Page 124: Fundamentals of Physical Geography: 11th NCERT

68 With reference to the proposed National Medical Commission Bill, consider the following statements.
1. The Bill proposes that the National Medical Commission (NMC) will have the authority to grant a
limited licence to certain mid-level practitioners connected with the modern medical profession to
practice medicine.
2. The Bill proposes to hold the National Exit Test for the students graduating from medical
institutions to obtain the licence for the practice.

Select the correct answer using the codes below.


A. 1 only
B. 2 only
C. Both 1 and 2
D. None of the above

Correct Answer : C

Answer Justification :

Justification: The National Medical Commission Bill seeks to improve the medical education
system in the country by ensuring availability of adequate and high-quality medical professionals,
periodic assessment of medical institutions, adoption of the latest medical research by medical
professionals and an effective grievance redressal mechanism.

The Bill has the following key features:

1. The Bill proposes to set up a medical commission, both at the national and state level,

61
Total Marks : 200
Online Prelims TEST - 7 (TEXTBOOK)
( InsightsIAS Mock Test Series for UPSC Preliminary Exam 2020 )

within three years of the passage of the legislation.

2. The Bill also has a provision for setting up a Medical Advisory Council by the Centre. The
council will act as a channel through which the states/Union Territories can convey their
views and concerns to the NMC.

3. The legislation also talks of conducting a uniform National Eligibility-cum-Entrance Test


(NEET) for admission to under-graduate medical education in all medical institutions
regulated under the Bill.

4. The Bill proposes to hold the National Exit Test for the students graduating from medical
institutions to obtain the licence for the practice. The test will also allow students to take
admission into post-graduate courses at medical institutions under this legislation.

5. The Bill says that the NMC will have the authority to grant a limited licence to certain
mid-level practitioners connected with the modern medical profession to practice
medicine.

Q Source:
https://www.insightsonindia.com/2019/08/09/insights-daily-current-affairs-pib-09-august-2019/

69 With respect to the award of Bharat Ratna, consider the following statements.
1. Its award is limited to achievements in the arts, literature, science and public services.
2. There is no written provision that Bharat Ratna should be awarded to Indian citizens only.
3. The award cannot be used as a prefix or suffix to the recipient’s name.

Select the correct answer using the codes below.


A. 1, 2 and 3
B. 2 only
C. 1 and 3 only
D. 2 and 3 only

Correct Answer : D

Answer Justification :

Justification: The President of India, Shri Ram Nath Kovind, presented Bharat Ratna Awards
to Shri Nanaji Deshmukh (posthumously), Dr Bhupendra Kumar Hazarika (posthumously) and Shri
Pranab Mukherjee.

Key facts:

Bharat Ratna is the highest civilian award of the Republic of India. The provision of

62
Total Marks : 200
Online Prelims TEST - 7 (TEXTBOOK)
( InsightsIAS Mock Test Series for UPSC Preliminary Exam 2020 )

Bharat Ratna was introduced in 1954.


Eligibility: Any person without distinction of race, occupation, position or sex is eligible for
these awards.
There is no written provision that Bharat Ratna should be awarded to Indian citizens
only.
It is awarded in recognition of exceptional service/performance of the highest order
in any field of human endeavour.
The award was originally limited to achievements in the arts, literature, science and public
services but the government expanded the criteria to include “any field of human
endeavour” in December 2011.
In terms of Article 18 (1) of the Constitution, the award cannot be used as a prefix or suffix
to the recipient’s name. However, should an award winner consider it necessary, he/she may
use the expression in their biodata/letterhead/visiting card etc. to indicate that he/she is a
recipient of the award.

Q Source: As mentioned above

70 Which of the following are the salient features of fold Mountains?


1. They are least likely to have conical peaks.
2. They are created when large areas are broken and displaced vertically.
3. They must be associated with volcanism either from the mountain core or its vicinity.

Select the correct answer using the codes below.


A. 1 and 3 only
B. 1, 2 and 3
C. 2 only
D. None of the above

Correct Answer : D

Answer Justification :

Justification: Statement 1: The Himalayan Mountains and the Alps are young fold mountains with
rugged relief and high conical peaks. The Appalachians in North America and the Ural mountains in
Russia have rounded features and low elevation. They are very old fold mountains. Therefore,
statement 1 does not have general validity.

Statement 2: Block Mountains are created when large areas are broken and displaced vertically.
The uplifted blocks are termed as horsts and the lowered blocks are called graben. The Rhine valley
and the Vosges mountain in Europe are examples of such mountain systems. So, 2 is incorrect.

Statement 3: It is not necessary. The Aravali range in India is one of the oldest fold mountain
systems in the world yet not associated with volcanic activity.

Q Source: Revision previous test syllabus: 7th Geography NCERT

63
Total Marks : 200
Online Prelims TEST - 7 (TEXTBOOK)
( InsightsIAS Mock Test Series for UPSC Preliminary Exam 2020 )

71 Akshay Patra, an organization based in Bengaluru is today world’s largest (not-for-profit run) “X
programme” serving millions in government schools and government-aided schools. Identify X?

A. Access to education for the Differently Abled


B. Mid-day meal
C. Scholarship for Girl students
D. Sanitation and Hygiene

Correct Answer : B

Answer Justification :

Learning: It strives to fight issues like hunger and malnutrition in India, by implementing the Mid-
Day Meal Scheme in the government schools and government-aided schools. Akshaya Patra aims
not only to fight hunger but also to bring children to school.

Akshaya Patra is continuously leveraging technology to cater to millions of children. Its state-
of-the-art kitchens have become a subject of study and they attract curious visitors from
around the world.
In partnership with the Government of India; various State Governments, the inestimable
support from many businesses, philanthropic donors and well-wishers; it has grown from our
humble beginnings in the year 2000, serving just 1,500 children across 5 schools.
Today Akshaya Patra is the world’s largest (not-for-profit run) mid-day meal programme
serving wholesome food to over 1.6 million children from over 13,000 schools across 11 states
in India.

Q Source: Syllabus: Major NGOs

72 Consider the following statements.


“Time Release Study”
1. is a tool reference in the WTO Trade Facilitation Agreement (TFA)
2. is facilitated by the World Customs Organization (WCO)
3. finds bottlenecks and gives a metric of expected delay in the implementation of free trade
agreement related tariffs in the concerned countries

Select the correct answer using the codes below.


A. 1 only
B. 1 and 3 only
C. 1, 2 and 3
D. 1 and 2 only

Correct Answer : D

Answer Justification :

Justification: The World Customs Organization (WCO) Time Release Study is a strategic and

64
Total Marks : 200
Online Prelims TEST - 7 (TEXTBOOK)
( InsightsIAS Mock Test Series for UPSC Preliminary Exam 2020 )

internationally recognized tool to measure the actual time required for the release and/or
clearance of goods, from the time of arrival until the physical release of cargo.

Aim: To find bottlenecks in the trade flow process and taking necessary measures to improve the
effectiveness and efficiency of border procedures.

The WCO TRS is specifically referenced in Article 7.6 of the WTO Trade Facilitation
Agreement (TFA) as a tool for Members to measure and publish the average release time of goods.

Uses:

1. The TRS is being increasingly used by Members with respect to strategic planning and the
proper sequencing of TFA measures in accordance with their National Committees on Trade
Facilitation (NCTF).

2. In recent years, the tool has been capturing a lot of attention worldwide; the international
donor community and the WCO development partners are recommending it as a key
performance measure to assess, evaluate, and enhance the implementation of the WTO TFA.

Q Source:
https://www.insightsonindia.com/2019/08/02/insights-daily-current-affairs-pib-02-august-2019/

73 Consider the following about the Bay of Bengal Initiative for Multi-Sectoral Technical and Economic
Cooperation (BIMSTEC).
#00000
1. Nepal and Bhutan are members of the BIMSTEC.
2. It was established through the Bangkok Declaration.
3. It is an implicit free trade agreement among the member nations.

Select the correct answer using the codes below.


A. 1 and 2 only
B. 2 and 3 only
C. 3 only
D. 1, 2 and 3

Correct Answer : A

Answer Justification :

Justification: The Bay of Bengal Initiative for Multi-Sectoral Technical and Economic Cooperation
(BIMSTEC) is a regional organization comprising seven Member States lying in the littoral and
adjacent areas of the Bay of Bengal constituting a contiguous regional unity.

This sub-regional organization came into being in 1997 through the Bangkok Declaration. It
constitutes seven Member States: five deriving from South Asia, including Bangladesh, Bhutan,

65
Total Marks : 200
Online Prelims TEST - 7 (TEXTBOOK)
( InsightsIAS Mock Test Series for UPSC Preliminary Exam 2020 )

India, Nepal, Sri Lanka, and two from Southeast Asia, including Myanmar and Thailand.

Initially, the economic bloc was formed with four Member States with the acronym 'BIST-EC'
(Bangladesh, India, Sri Lanka and Thailand Economic Cooperation).

Later, Myanmar was included and it became BIMS-TEC.

It is not a FTA. IT is a sector-drive organization focussing on sharing best practices and help each
other in agriculture, public health, poverty alleviation, counter-terrorism, environment, culture etc.

Q Source: Syllabus: Asian intergovernmental bodies

74 Which of these have caused some of the gravest threat to mangroves?


1. Destruction of coral reefs
2. Diversion of fresh water sources away from the seas and oceans

Which of the above is/are correct?


A. 1 only
B. 2 only
C. Both 1 and 2
D. None

Correct Answer : C

Answer Justification :

Justification: Mangrove forests have reached a critical stage of decline in many parts of the world.

Statement 1: Coral reefs provide the first barrier against currents and strong waves. When they are
destroyed, the stronger-than-normal waves and currents reaching the coast can undermine the fine
sediment in which the mangroves grow. This can prevent seedlings from taking root and wash away
nutrients essential for mangrove ecosystems.

Statement 2: River changes are caused due to dams and irrigation reducing the amount of water
reaching mangrove forests. This changes the salinity level of water in the forest. If salinity becomes
too high, the mangroves cannot survive. Freshwater diversions can also lead to mangroves drying
out.

Oil pollution has also led to the threat; it can smother mangrove roots and suffocate the trees.

You can read more here


http://wwf.panda.org/about_our_earth/blue_planet/coasts/mangroves/mangrove_threats/

Q Source: Fundamentals of Physical Geography: 11th NCERT

75 Consider the following statements.


1. Minerals on earth mainly come from Magma.

66
Total Marks : 200
Online Prelims TEST - 7 (TEXTBOOK)
( InsightsIAS Mock Test Series for UPSC Preliminary Exam 2020 )

2. Magma cools slowly as it rises towards Earth’s surface.

Which of the above is/are correct?


A. 1 only
B. 2 only
C. Both 1 and 2
D. None of the above

Correct Answer : C

Answer Justification :

Learning: Magma cools slowly as it rises towards Earth’s surface. It can take thousands to millions
of years to become solid when it is trapped inside Earth.

As the magma cools, solid rocks form (igneous).


These rocks are mixtures of minerals. Granite is a common rock that forms when magma
cools. Granite contains the minerals quartz, plagioclase feldspar, and potassium feldspar.
The same igneous rock can be eroded, weathered, transported, deposited and consolidated as
sedimentary rocks which are the source of non-metallic minerals.
Moreover, fossils are formed by decomposition of organic material in situ. After long time
they get converted in useful hydrocarbons like petroleum and coal.

Q Source: Fundamentals of Physical Geography: 11th NCERT

76 Consider the following statements.


1. According to the latest tiger census, India’s tiger population has doubled in the past dozen years.
2. About only 30 percent of the world’s tiger population now resides in India.
3. M-STrIPES is a GPS-enabled application used by forest guards to help capture tiger related
statistics.
4. Tigers in India can be found in Shivalik Hills and Gangetic Plains.

Select the correct answer using the codes below.


A. 1, 3 and 4 only
B. 1, 2 and 3 only
C. 2 and 4 only
D. 1, 2, 3 and 4

Correct Answer : A

Answer Justification :

Justification: Statement 1: According to the latest tiger census, India’s tiger population has
doubled in the past dozen years, a significant achievement for the country’s wildlife conservation
efforts.

67
Total Marks : 200
Online Prelims TEST - 7 (TEXTBOOK)
( InsightsIAS Mock Test Series for UPSC Preliminary Exam 2020 )

Releasing results of the 2018 census, Prime Minister Narendra Modi said India was now “one of
the biggest and most secure habitats of the tiger.” India estimates that over 75 percent of
the world’s tiger population now resides in the country.

India is now home to 2,967 tigers, up from 1,411 in 2006 when it conducted its first national
survey. The last census in 2014 had counted 2,226 tigers.

The uptick in the tiger population is good news for India, which has grappled with human-wildlife
conflict amid rapid urbanization.

The highest number of tigers is in Madhya Pradesh in central India, which has 526. The tiger
is India’s national animal.

Statement 3 and 4: The M-STrIPES, the application used by forest guards, is GPS-enabled and helps
to capture data relating to tiger sightings, deaths, wildlife crime and ecological observations while
patrolling.Since tigers keep moving between states, conservationists prefer to talk about tiger
numbers in terms of landscapes.

The biggest increase has been in Madhya Pradesh a massive 218 individuals (71%) from 308 in
2014 to 526.

In Maharashtra, the number has gone up from 190 to 312 (64%), and in Karnataka, from 406 to 524
(118, or 29%). Uttarakhand has gained over 100 tigers (340 to 442; 30%)

India’s five tiger landscapes are: Shivalik Hills and Gangetic Plains, Central Indian Landscape and
Eastern Ghats, Western Ghats, North-East Hills and Brahmaputra Plains, and the Sundarbans.

Q Source:
https://www.insightsonindia.com/2019/08/01/insights-into-editorial-indias-tiger-population-doubles-i
n-a-dozen-years/

77 Consider the following about Anticyclones.


1. These are low pressure regions and their extent is always small.
2. Generally shows clear and fine weather.

Select the correct answer using the codes below.


A. 1 only
B. 2 only
C. Both 1 and 2
D. None of the above

Correct Answer : B

Answer Justification :

Justification: Statement 1: An anti-cyclone -- also known as a high pressure area -- is a large


atmospheric circulation system with the wind flowing clockwise around it in the Northern

68
Total Marks : 200
Online Prelims TEST - 7 (TEXTBOOK)
( InsightsIAS Mock Test Series for UPSC Preliminary Exam 2020 )

Hemisphere, and counter-clockwise in the Southern Hemisphere.

The extents are generally large and the effects are also pronounced.

Statement 2: Anticyclones form from air masses cooling more than their surroundings, which
causes the air to contract slightly making the air denser. Since dense air weighs more, the weight of
the atmosphere overlying a location increases, causing increased surface air pressure.

Winds are of moderate speed, and at the outer edges, cloudy and precipitation conditions exist.

Please see this http://tinyurl.com/j5edaou

Cloudy conditions and precipitation exists at the periphery of anticyclones. Otherwise, such a high
pressure region generally shows fine and clear weather.

Q Source: AR: 11th NCERT: Fundamentals of Physical Geography

78 Members of the Asia-Pacific Economic Cooperation (APEC) include


1. Singapore
2. USA
3. China
4. India

Select the correct answer using the codes below.


A. 1 and 2 only
B. 1, 2 and 3 only
C. 1, 2 and 4 only
D. 3 and 4 only

Correct Answer : B

Answer Justification :

Justification: Asia-Pacific Economic Cooperation (APEC) is an inter-governmental forum for 21


Pacific Rim member economies that promotes free trade throughout the Asia-Pacific region.
Inspired from the success of Association of Southeast Asian Nations (ASEAN)’s series of post-
ministerial conferences launched in the mid-1980s, the APEC was established in 1989 in response to
the growing interdependence of Asia-Pacific economies and the advent of regional trade blocs in
other parts of the world; and to establish new markets for agricultural products and raw materials
beyond Europe.

Headquartered in Singapore, the APEC is recognized as one of the highest-level multilateral blocs
and oldest forums in the Asia-Pacific region, and exerts a significant global influence.

APEC ensures that goods, services, investment and people move easily across borders. Members
facilitate this trade through faster customs procedures at borders; more favorable business climates
behind the border; and aligning regulations and standards across the region. For example, APEC's
initiatives to synchronize regulatory systems is a key step to integrating the Asia-Pacific economy. A
69
Total Marks : 200
Online Prelims TEST - 7 (TEXTBOOK)
( InsightsIAS Mock Test Series for UPSC Preliminary Exam 2020 )

product can be more easily exported with just one set of common standards across all economies.

Q Source: https://www.apec.org/About-Us/About-APEC

79 The thickness of the troposphere is greater at the equator rather than at the poles because of
1. Convection currents leading to thermal expansion of the atmosphere at the equator
2. Rotation of the earth pushes the atmosphere near equator to greater heights due to centrifugal
force

Which of the above is/are correct?


A. 1 only
B. 2 only
C. Both 1 and 2
D. None of the above

Correct Answer : C

Answer Justification :

Justification: The troposphere is thicker at the equator than at the poles because the equator is
warmer. The convection currents of air expand the thickness of the troposphere (atmosphere).

Thus the simple reason is thermal expansion of the atmosphere at the equator and thermal
contraction near the poles.

Also, the rotation of the earth causes centrifugal force which is strongest near the equator and
pushes the atmosphere to greater heights.

The thickness of the troposphere also varies with season. The troposphere is thicker in the summer

70
Total Marks : 200
Online Prelims TEST - 7 (TEXTBOOK)
( InsightsIAS Mock Test Series for UPSC Preliminary Exam 2020 )

and thinner in the winter all around the planet.

At the poles in winter, the atmosphere is uniformly very cold and the troposphere cannot be
distinguished from other layers.

Q Source: Additional Research: NCERT 11th Fundamentals of Physical Geography

80 Aerobic rice cultivation is to be preferred over anaerobic rice cultivation where


1. Soil contains poor moisture.
2. No chemical fertilizer can be added to the soil.

Which of the above is/are correct?


A. 1 only
B. 2 only
C. Both 1 and 2
D. None

Correct Answer : A

Answer Justification :

Concept: Aerobic rice cultivation is growing rice plant as irrigated crop like cultivating maize and
wheat in aerobic condition, where oxygen is plenty in soil.

Throughout the growing season, aerobic rice field is kept under unsaturated condition and
field is irrigated by surface or sprinkler system to keep soil wet. Therefore, water productivity
is reported to be higher in aerobic rice.
In aerobic rice cultivation, rice is cultivated as direct sown in nonpuddle aerobic soil under
supplementary irrigation and fertiliser with suitable high yielding rice varieties.

Justification: The suitable areas for aerobic rice cultivation includes irrigated lowlands, where
rainfall is insufficient to sustain rice production, delta regions where there is delay in water release
from reservoir, irrigated system of rice cultivation, where pumping from deep bore well has become
so expensive and favourable upland system has access to supplementary irrigation.

Accordingly, Tamil Nadu, Jharkhand, Chhattisgarh, parts of Bihar, Odisha, Karnataka, and
eastern Uttar Pradesh are the projected area where there is uneven distribution and frequent
occurrence of soil moisture limitation.
Aerobic rice cultivation needs suitable rice varieties having the characteristics of both upland
and high yielding lowland varieties to get good yield under the new unconventional system of
cultivation.
This system also involves mechanised way of sowing with no puddling, transplanting and no
need of frequent irrigation, which reduce labour usage more than 50%, compared to irrigated
rice.

Q Source: Surprise questions: http://irri.org/our-work/research/better-rice-varieties/aerobic-rice

71
Total Marks : 200
Online Prelims TEST - 7 (TEXTBOOK)
( InsightsIAS Mock Test Series for UPSC Preliminary Exam 2020 )

81 Consider the following statements.


1. Based on recent reports, most of the cases of mob lynching reported in the country have happened
in the state of Uttar Pradesh.
2. The Indian Penal Code and the Criminal Procedure Code have no provisions to deal with the cases
of mob lynching incidents.

Which of the above is/are correct?


A. 1 only
B. 2 only
C. Both 1 and 2
D. None of the above

Correct Answer : D

Answer Justification :

Justification: While the Indian Penal Code and the Criminal Procedure Code have provisions to
deal with the cases of mob lynching incidents, they are not adequate. Accordingly, the government
has brought the Bill to provide for stricter punishment to curb such incidents.

• After 2014, 86% cases of mob lynching reported in the country happened in Rajasthan.

• Legislation fixes command responsibility for communal incidents. It recognises that targeted
communal violence disproportionately victimises minorities and it creates a mechanism to insulate
investigations of communal violence from political interference.

Learning: Rajasthan Assembly has passed the Anti-Lynching Bill.

SC guidelines on mob-lynching:

1. There shall be a “separate offence” for lynching and the trial courts must ordinarily award
maximum sentence upon conviction of the accused person to set a stern example in cases of
mob violence.

2. The state governments will have to designate a senior police officer in each district
for taking measures to prevent incidents of mob violence and lynching.

3. The state governments need to identify districts, sub-divisions and villages where
instances of lynching and mob violence have been reported in the recent past.

4. The nodal officers shall bring to the notice of the DGP about any inter-district co-ordination
issues for devising a strategy to tackle lynching and mob violence related issues.

5. Every police officer shall ensure to disperse the mob that has a tendency to cause

72
Total Marks : 200
Online Prelims TEST - 7 (TEXTBOOK)
( InsightsIAS Mock Test Series for UPSC Preliminary Exam 2020 )

violence in the disguise of vigilantism or otherwise.

6. Central and the state governments shall broadcast on radio, television and other media
platforms about the serious consequences of mob lynching and mob violence.

7. Despite the measures taken by the State Police, if it comes to the notice of the local police
that an incident of lynching or mob violence has taken place, the jurisdictional police station
shall immediately lodge an FIR.

8. The State Governments shall prepare a lynching/mob violence victim compensation


scheme in the light of the provisions of Section 357A of CrPC within one month from the date
of this judgment.

9. If a police officer or an officer of the district administration fails to fulfill his duty, it will be
considered an act of deliberate negligence.

Q Source:
https://www.insightsonindia.com/2019/08/06/insights-daily-current-affairs-pib-06-august-2019/

82 There is a growing consciousness of climate change in trade and commerce. In the context, consider
the following statements.
Assertion (A): There are no WTO rules specific to climate change.
Reason (R): Trade is not related to carbon emissions.

In the context of the above, which of these is correct?


A. A is correct, and R is an appropriate explanation of A.
B. A is correct, but R is not an appropriate explanation of A.
C. A is correct, but R is incorrect.
D. Both A and R are incorrect.

Correct Answer : C

Answer Justification :

Justification: As per WTO’s website, “The issue of climate change, per se, is not part of the WTO's
ongoing work programme and there are no WTO rules specific to climate change. However, the
WTO is relevant because climate change measures and policies intersect with international trade in
a number of different ways.”

However, the WTO offers a powerful supporting framework for sustainable development and green
economy.

It provides an enabling environment through its objectives, institutions and monitoring of potential

73
Total Marks : 200
Online Prelims TEST - 7 (TEXTBOOK)
( InsightsIAS Mock Test Series for UPSC Preliminary Exam 2020 )

trade protectionism, enforcement mechanism, toolbox of rules, and growing case law in the
environment area.

Q Source: Surprise questions: https://www.wto.org/english/tratop_e/envir_e/envir_e.htm

83 Arrange these geographical regions from the West to east.


1. Gulf of Aden
2. Laccadive Sea
3. Gulf of Thailand
4. South China Sea

Select the correct answer using the codes below.


A. 1234
B. 2143
C. 2134
D. 1243

Correct Answer : A

Answer Justification :

Learning:

Q Source: Map-based: Asia

84 Which of the following fall under the Peninsular block of India?


1. Karbi Anglong Plateau
2. Rajmahal Hills
3. Meghalaya Plateau

74
Total Marks : 200
Online Prelims TEST - 7 (TEXTBOOK)
( InsightsIAS Mock Test Series for UPSC Preliminary Exam 2020 )

Select the correct answer using the codes below:


A. 1 and 2 only
B. 2 and 3 only
C. 1 only
D. 1, 2 and 3

Correct Answer : D

Answer Justification :

Learning: The northern boundary of the Peninsular Block may be taken as an irregular line running
from Kachchh along the western flank of the Aravali Range near Delhi and then roughly parallel to
the Yamuna and the Ganga as far as the Rajmahal Hills and the Ganga delta.

Apart from these, the Karbi Anglong and the Meghalaya Plateau in the northeast and Rajasthan in
the west are also extensions of this block. The northeastern parts are separated by the Malda fault
in West Bengal from the Chotanagpur plateau. In Rajasthan, the desert and other desert–like
features overlay this block.

Q Source: Revision: 9th NCERT Geography

85 With reference to Partnerships in Environmental Management for the Seas of East Asia (PEMSEA),
consider the following statements.
1. It is a regional partnership programme implemented by the World bank.
2. The areas of work under it include, inter alia, coastal and ocean governance, natural and man-made
hazard prevention and management.
3. India is a partner country.

Select the correct answer using the codes below.


A. 1 and 3 only
B. 2 only
C. 1 and 2 only
D. 1, 2 and 3

Correct Answer : B

Answer Justification :

Justification: Statement 1: Partnerships in Environmental Management for the Seas of East Asia or
PEMSEA is a regional partnership programme implemented by the United Nations Development
Programme (UNDP) and executed by the United Nations Office for Project Services (UNOPS). The
project, started in 1994, was originally known as Prevention and Management of Marine Pollution in
the East Asian Seas (SDS-SEA).

PEMSEA is currently being hosted by the Philippines' Department of Environment and Natural
Resources and holds its office in the DENR compound in Quezon City, Philippines.

75
Total Marks : 200
Online Prelims TEST - 7 (TEXTBOOK)
( InsightsIAS Mock Test Series for UPSC Preliminary Exam 2020 )

Statement 2: PEMSEA's areas of work include coastal and ocean governance, natural and man-
made hazard prevention and management, habitat protection, restoration and management, water
use and supply management, pollution and waste reduction management, as well as food security
and livelihood management.

One of the important tasks that PEMSEA assumes is turning the knowledge about regional coastal
into action that can improve the status quo. PEMSEA capitalizes on its broad intergovernmental,
financial and intellectual resources to come up with the best solutions problems of sustainable
coastal management.

PEMSEA's partners include the following countries and organizations:

Country Partners
Cambodia
China
Indonesia
Japan
Laos
North Korea
Philippines
South Korea
Singapore
Thailand
Timor-Leste
Vietnam
Q Source:
https://en.wikipedia.org/wiki/Partnerships_in_Environmental_Management_for_the_Seas_of_E
ast_Asia

86 Conditions that favour the formation of deltas include


1. Coasts without shelters with a large tidal variation
2. Frequent water diversions from the main river that forms the delta
3. Presence of Continental shelf

Select the correct answer using the codes below.


A. 1 and 2 only
B. 3 only
C. 2 and 3 only
D. 1 and 3 only

Correct Answer : B

Answer Justification :

Justification: Conditions Favourable for Delta Formation

Active erosion of the river in its upper course to provide extensive gravel, sand and silt to be
eventually deposited as deltas.

76
Total Marks : 200
Online Prelims TEST - 7 (TEXTBOOK)
( InsightsIAS Mock Test Series for UPSC Preliminary Exam 2020 )

The coast should be sheltered preferably tideless, else delta will be washed away.
There should be no strong current running at right angle to the river mouth, as it can wash
away the sediments.
Any large lake in the way or river course can filter off the sediments, thus unfavourable for
delta formation.
The sea should be shallow adjoining the delta as the sediments will disappear in the deep
waters of the sea. This is why a continental shelf is preferred.

Q Source: AR: Fundamentals of Physical Geography

87 Why are estuaries considered favourable for situating ports?


1. Open connection to the Sea or Ocean.
2. Depth of water for anchoring and stationing ships.

Which of the above is/are correct?


A. 1 only
B. 2 only
C. Both 1 and 2
D. None

Correct Answer : C

Answer Justification :

Justification: An estuary is a partially enclosed coastal body of brackish water with one or more
rivers or streams flowing into it, and with a free connection to the open sea.

They are an important part of the shipping industry because there are many industrial ports located
in estuaries due its depth of water and connection with open sea.

As against estuaries, deltas are not preferred for coastal shipping because deltas do not offer the
depth of water required for shipping and may not offer the required connection to the open sea.

Q Source: Chapter 5: Goh Cheng Leong - Certificate Physical and Human Geography

88 Consider the following about the Bharhut Stupa.


1. While built during the Ashokan period, the Shungas improvised the art work at Bharhut.
2. Queen Maya’s dream and some Jatakas have been depicted at the stupa.

Select the correct answer using the codes below.


A. 1 only
B. 2 only
C. Both 1 and 2
D. None of the above

77
Total Marks : 200
Online Prelims TEST - 7 (TEXTBOOK)
( InsightsIAS Mock Test Series for UPSC Preliminary Exam 2020 )

Correct Answer : C

Answer Justification :

Justification: Statement 1: The Bharhut stupa (in MP) may have been first built by the Maurya king
Ashoka in the 3rd century BCE, but many works of art were apparently added during the Shunga
period, with many friezes from the 2nd century BCE.

Statement 2: They contain numerous birth stories of the Buddha's previous lives, or Jataka tales.
Many of them are in the shape of large, round medallions.

Queen Maya’s dream, preceding the birth of the Buddha, is also a major theme on the railing of the
Bharhut “stupa”.

Q Source: Revision: Chapter 4: 11th NCERT: An Introduction to Indian Arts

89 In an urban area affected by traffic jams, which of the following measures can cut down the level of
pollution in the area?
1. Planting juniper and pinus trees in such areas
2. Fitting catalytic converters in automobile exhausts

Which of the above is/are correct?


A. 1 only
B. 2 only
C. Both 1 and 2
D. None

Correct Answer : C

Answer Justification :

78
Total Marks : 200
Online Prelims TEST - 7 (TEXTBOOK)
( InsightsIAS Mock Test Series for UPSC Preliminary Exam 2020 )

Justification: Statement 1: Certain plants e.g., Pinus, Juniparus, Quercus, Pyrus and Vitis can
metabolise nitrogen oxide and therefore, their plantation could help in this matter.

Statement 2: Many techniques are used to control or reduce the formation of photochemical smog.
If we control the primary precursors of photochemical smog, such as NO2 and hydrocarbons, the
secondary precursors such as ozone and PAN, the photochemical smog will automatically be
reduced.

Usually catalytic converters are used in the automobiles, which prevent the release of nitrogen
oxide and hydrocarbons to the atmosphere. They convert it into simpler compounds and thus reduce
the pollution.

Q Source: Page 404: Chemistry Unit 14: XIth NCERT

90 With respect to the Transgender Persons (Protection of Rights) Bill 2019, consider the following
statements.
1. The bill requires transgender persons to go through a district magistrate and “district screening
committee” to get certified as a transperson.
2. The Bill provides reservations to transgender persons in certain sectors.
3. The bill allows organised begging by transgenders upon certification by a district magistrate.

Select the correct answer using the codes below.


A. 2 only
B. 1 and 3 only
C. 1, 2 and 3 only
D. 1 only

Correct Answer : D

Answer Justification :

Justification: Lok Sabha approves The Transgender Persons (Protection of Rights) Bill 2019.

New definition:

According to the new definition, a transgender person is somebody “whose gender does not match
the gender assigned to that person at birth and includes trans-men or trans-women, persons with
intersex variations, gender-queers, and persons having socio-cultural identities such as kinnar,
hijras, aravani, and jogta”.

Highlights of the Bill:

1. The Bill aims to stop discrimination against a transgender person in various sectors such as
education, employment, and healthcare. It also directs the central and state governments to
provide welfare schemes for them.

79
Total Marks : 200
Online Prelims TEST - 7 (TEXTBOOK)
( InsightsIAS Mock Test Series for UPSC Preliminary Exam 2020 )

2. The Bill states that a person will be recognised as transgender on the basis of a certificate of
identity issued through the district screening committee. This certificate will be a proof of
identity as transgender and confer rights under this Bill.

3. Going by the bill, a person would have the right to choose to be identified as a man, woman or
transgender, irrespective of sex reassignment surgery and hormonal therapy.

4. It also requires transgender persons to go through a district magistrate and “district


screening committee” to get certified as a transperson.

5. The committee would comprise a medical officer, a psychologist or psychiatrist, a district


welfare officer, a government official, and a transgender person.

Criticisms:

1. The Bill is silent on granting reservations to transgender persons.

2. The bill has prescribed punishments for organised begging. However, the Bill doesn’t provide
anything to better to condition in those areas, it doesn’t provide for reservation.

3. The Transgender Bill does not mention any punishments for rape or sexual assault of
transgender persons as according to Sections 375 and 376 of the Indian Penal Code, rape is
only when a man forcefully enters a woman.

Q Source:
https://www.insightsonindia.com/2019/08/06/insights-daily-current-affairs-pib-06-august-2019/

91 The Reserve Bank of India (RBI) has tightened the fit-and-proper criteria for directors on the boards
of state-run banks. As per the norms proposed recently by the RBI
1. The candidate should not be ordinarily connected with para-banking activities
2. No person is to be elected or re-elected to a bank board if the candidate has served as a director in
the past on the board of any bank or the RBI or insurance company for a certain duration of time
3. The candidate should not be engaging in the business of stock broking.
4. The candidate should not be a member of Parliament, state legislature or a municipal corporation

Select the correct answer using the codes below.


A. 2 and 3 only
B. 1, 2 and 3 only
C. 1 and 4 only
D. 1, 2, 3 and 4

80
Total Marks : 200
Online Prelims TEST - 7 (TEXTBOOK)
( InsightsIAS Mock Test Series for UPSC Preliminary Exam 2020 )

Correct Answer : D

Answer Justification :

Justification: The revised norms are applicable only to public sector banks (PSBs).

Key changes proposed:

As per the Reserve Bank of India (‘Fit and Proper’ Criteria for Elected Directors on
the Boards of PSBs) Directions, 2019, all the banks — SBI and nationalised banks — are
required to constitute a Nomination and Remuneration Committee (NRC).

Centre’s nominee director shall not be part of the nomination and remuneration
committee (NRC).

The terms with regard to the NRC and the manner of the appointment of directors have
been aligned with the practice in private banks, the recommendations made by
the Banks Board Bureau, and with the provisions in the Companies Act.

Composition of NRC: The NRC will have a minimum of three non-executive directors from
amongst the board of directors. Of this, not less than one-half shall be independent directors
and should include at least one member from the risk management committee of the board.

Eligibility: As per the directions, the candidate who wants to become an elected director
should at least be a graduate. He/She should be between 35-67 years old as on the cut-off
date fixed for submission of nominations for election. The candidate should have special
knowledge or practical experience in areas useful for banks.

An elected director shall hold office for three years and shall be eligible for re-election,
provided that no director hold office for a period exceeding six years, whether served
continuously or intermittently.

What will also be under scrutiny is the ‘list of entities’ in which a prospective director has an
interest – to ascertain if such a firm is in default or has been in default in the past decade.

The negative list says that:

1. The candidate should not be a member of the board of any bank, the RBI, financial institution
(FI), insurance company or a non-operative financial holding company (NOFHC).

2. The candidate should not be connected with hire-purchase, financing, money lending,

81
Total Marks : 200
Online Prelims TEST - 7 (TEXTBOOK)
( InsightsIAS Mock Test Series for UPSC Preliminary Exam 2020 )

investment, leasing and other para-banking activities. But “investors of such entities would
not be disqualified for appointment as directors if they do not enjoy any managerial control in
them”.

3. No person is to be elected or re-elected to a bank board if the candidate has served as a


director in the past on the board of any bank, the RBI or insurance company under any
category for six years, whether continuously or intermittently.

4. The candidate should not be engaging in the business of stock broking.

5. The candidate should not be a member of Parliament, state legislature, municipal corporation,
municipality, or other local bodies — notified area council, city council, panchayat, gram
sabha or zila parishad.

6. Other conditions are that candidate should not be a partner of a chartered accountant (CA)
firm currently engaged as a statutory central auditor of any nationalised bank or State Bank
of India; or when the firm is engaged as statutory branch auditor or concurrent auditor of the
bank in which nomination is sought.

Q Source:
https://www.insightsonindia.com/2019/08/05/insights-daily-current-affairs-pib-05-august-2019/

92 Phytoplankton are generally found in upper layers of ocean water due to


1. Absence of nutrients in deep ocean water
2. Sunlight abundance in top layers

Select the correct answer using the codes below.


A. 1 only
B. 2 only
C. Both 1 and 2
D. None of the above

Correct Answer : B

Answer Justification :

Justification: Phytoplankton in the open ocean need both sunlight and nutrients (such as nitrate
and phosphate) to be able to photosynthesize.

Sunlight is only available in the uppermost layers.


During photosynthesis, the nutrients are quickly used up by phytoplankton so they are not
available for long periods in the upper layers under normal circumstances.

82
Total Marks : 200
Online Prelims TEST - 7 (TEXTBOOK)
( InsightsIAS Mock Test Series for UPSC Preliminary Exam 2020 )

This is indeed the case in tropical waters, and as a result they are very unproductive.

To escape this problem the seawater needs to be mixed regularly to bring the nutrient rich
deep waters up to the sunlight zone where the phytoplankton can grow.

This is one of the reasons why areas where cold and warm currents meet are very productive; the
collision of currents causes mixing.

Q Source: Page 124: Fundamentals of Physical Geography: 11th NCERT

93 With reference to South Asian Association for Regional Cooperation (SAARC), consider the following
statements.
1. Its secretariat is based in India.
2. SAARC is an observer at the United Nations.
3. One of its objectives as mentioned in the SAARC Charter is “to cooperate with international and
regional organizations with similar aims and purposes”.
4. Decisions at all levels are to be taken on the basis of unanimity; and bilateral and contentious issues
are excluded from the deliberations of the Association.

Select the correct answer using the codes below.


A. 1, 2, 3 and 4
B. 2, 3 and 4 only
C. 3 only
D. 1, 2 and 4 only

Correct Answer : B

Answer Justification :

Justification: Statement 1: SAARC is the regional intergovernmental organization and geopolitical


union of states in South Asia. Its member states include Afghanistan, Bangladesh, Bhutan, India, the
Maldives, Nepal, Pakistan and Sri Lanka.

SAARC was founded in Dhaka on 8 December 1985. Its secretariat is based in Kathmandu, Nepal.
The organization promotes development of economic and regional integration. It launched the
South Asian Free Trade Area in 2006.

Statement 2: SAARC maintains permanent diplomatic relations at the United Nations as an observer
and has developed links with multilateral entities, including the European Union.

Statement 3: The objectives of the Association as outlined in the SAARC Charter are: to promote the
welfare of the peoples of South Asia and to improve their quality of life; to accelerate economic
growth, social progress and cultural development in the region and to provide all individuals the
opportunity to live in dignity and to realize their full potentials; to promote and strengthen
collective self-reliance among the countries of South Asia; to contribute to mutual trust,
understanding and appreciation of one another's problems; to promote active collaboration and

83
Total Marks : 200
Online Prelims TEST - 7 (TEXTBOOK)
( InsightsIAS Mock Test Series for UPSC Preliminary Exam 2020 )

mutual assistance in the economic, social, cultural, technical and scientific fields; to strengthen
cooperation with other developing countries; to strengthen cooperation among themselves in
international forums on matters of common interests; and to cooperate with international and
regional organizations with similar aims and purposes.

Statement 4: Decisions at all levels are to be taken on the basis of unanimity; and bilateral and
contentious issues are excluded from the deliberations of the Association.

94 Consider the following statements.


Bajada are
1. Consecutive series of alluvial fans forming along the edge of a linear mountain range.
2. Usually occur in arid climates.

Which of the above is/are correct?


A. 1 only
B. 2 only
C. Both 1 and 2
D. None of the above

Correct Answer : C

Answer Justification :

Learning: A bajada (one of the depositional forms in arid areas) consists of a series of coalescing
alluvial fans along a mountain front. These fan-shaped deposits form from the deposition of
sediment within a stream onto flat land at the base of a mountain.

Surface of this feature undulates in a rolling fashion as one moves from the centre of one alluvial
fan to another.

84
Total Marks : 200
Online Prelims TEST - 7 (TEXTBOOK)
( InsightsIAS Mock Test Series for UPSC Preliminary Exam 2020 )

You can read about other forms here https://www.nps.gov/subjects/geology/arid-landforms.htm

Q Source: AR: Glossary: Fundamentals of Physical Geography: 11th NCERT

95 “Asian Development Outlook” report is published by

A. Asia Pacific and Economic Cooperation (APEC)


B. Asian Infrastructure and Investment Bank (AIIB)
C. Asian Development Bank (ADB)
D. None of the above

Correct Answer : C

Answer Justification :

Justification: The Asian Development Outlook analyzes economic and development issues in
developing countries in Asia. This includes forecasting the inflation and gross domestic product
growth rates of countries throughout the region, including the People's Republic of China and India.

Growth in Developing Asia is projected to expand by 5.7% in 2019 despite the trade conflict
between the People’s Republic of China (PRC) and the United States (US).

Developing Asia is forecast to maintain strong but moderating growth in line with April forecasts in
Asian Development Outlook 2019 (ADO 2019), even as trade conflict continues between the PRC
and the US. The regional gross domestic product (GDP) is forecast to expand by 5.7% in 2019, as
unexpectedly strong growth in Central Asia offsets small downgrades for East, South, and Southeast
Asia in 2019, with growth slowing marginally to 5.6% in 2020.

Q Source: https://www.adb.org/publications/ado-supplement-july-2019

96 Consider the following statements.


1. Sunspots are temporary phenomena on the photosphere of the Sun.
2. They appear visibly as bright spots compared to surrounding regions.
3. They may affect the climate on earth by changing the solar output received by earth.

Select the correct answer using the codes below.


A. 1 only
B. 2 and 3 only
C. 1 and 3 only
D. 1, 2 and 3

Correct Answer : C

Answer Justification :

85
Total Marks : 200
Online Prelims TEST - 7 (TEXTBOOK)
( InsightsIAS Mock Test Series for UPSC Preliminary Exam 2020 )

Justification: They appear as darker spots. They reduce solar output if they increase in number.

Moreover, they reflect intense magnetic activity on the Sun at those spots.

If the solar output is changed, it would affect the temperature and wind patterns on earth causing
either a short-term weather change or if persistent may also cause a long-term climate change.

Q Source: Page 107: Fundamentals of Physical Geography: 11th NCERT

97 Which of following characterizes the Western coasts of India?


1. Highly indented

A. Highly indented
B. Formation of wave-cut platforms in sea cliffs
C. Lagoons, deltas and swamps
D. Both (a) and (b)

Correct Answer : D

Answer Justification :

Learning: The west coast of our country is a high rocky retreating coast. On the other hand,
erosional forms dominate in the west coast instead of depositional forms.

High rocks coast features:

Highly indented
Formation of wave-cut platforms in front of the sea cliff due to constant water erosional action
Material eroded in such manner, deposits along the off-shore forming wave terraces.
Several depositions like this results in the formation of barrier bars and spits that break block
the sea water forming a lagoon.

Low Sedimentary coast features:

Lagoons, deltas etc. found


Lagoons eventually turn into swamps which turn into coastal plains
Storm and tsunami waves cause darastic changes in the supply of depositional material

Q Source: Revision: Chapter 7: Fundamentals of Physical Geography: 11th NCERT

98 Which of the following gases contribute to global warming?


1. Methane
2. Water Vapour
3. Nitrogen
4. Ozone
5. HFC-23

86
Total Marks : 200
Online Prelims TEST - 7 (TEXTBOOK)
( InsightsIAS Mock Test Series for UPSC Preliminary Exam 2020 )

Select the correct answer using the codes below.


A. 1, 2 and 4 only
B. 1, 2, 4 and 5 only
C. 3 and 5 only
D. 1, 2, 3, 4 and 5

Correct Answer : B

Answer Justification :

Justification: Statement 1, 2 and 4: Besides carbon dioxide, other greenhouse gases are methane,
water vapour, nitrous oxide, CFCs and ozone. Chlorofluorocarbons (CFCs) are man-made industrial
chemicals used in air conditioning etc. CFCs are also damaging the ozone layer.

Statement 3: It is not a greenhouse gas, but NOx are GHGs. similar to how oxygen is not a GHG, but
ozone is.

Statement 5: India recently announced that it will eliminate the HFC-23 gas, a potent greenhouse
gas with high global warming potential, as part of its commitment to combat the threat emanating
from climate-damaging HFCs.

http://economictimes.indiatimes.com/news/politics-and-nation/india-to-eliminate-potent-greenhouse-
gas-hfc-23/articleshow/54834662.cms

Q Source: Current affairs + AR: Page 401: Chemistry Unit 14: XIth NCERT

99 Which of the following about the tectonic ‘Indian Plate’ is NOT correct?

A. It is a major tectonic plate.


B. It forms a convergent late boundary with the Himalayas.
C. The Indian plate is tectonically separated from the Peninsular India plate.
D. The plate extends to Pakistan and Myanmar as well.

Correct Answer : C

Answer Justification :

Justification: Plates are divided between major and minor plates based on their geographical
coverage. Indian plate is thus a major plate. So, (a) is correct.

The subduction zone along the Himalayas forms the northern plate boundary in the form of
continent— continent convergence. So, (b) is correct.

Peninsular plate is an integral part of the Indian plate. So, (c) is incorrect.

In the east, it extends through Rakinyoma Mountains of Myanmar towards the island arc along the

87
Total Marks : 200
Online Prelims TEST - 7 (TEXTBOOK)
( InsightsIAS Mock Test Series for UPSC Preliminary Exam 2020 )

Java Trench. The Western margin follows Kirthar Mountain of Pakistan. So, (d) is correct.

Q Source: Revision: Chapter 4: Fundamentals of Physical Geography: 11th NCERT

100 Which of these major regions of the world are Coffee growing belts?
1. Latin America
2. Africa
3. South and South East Asia

Select the correct answer using the codes below.


A. 1 and 2 only
B. 2 and 3 only
C. 3 only
D. 1, 2 and 3

Correct Answer : D

Answer Justification :

Justification: The image contains required details.

Q Source: Revision: 9th NCERT Geography

88

S-ar putea să vă placă și